KM - StrengthenKM - Strengthen and Weaken - PDF and Weaken

July 13, 2022 | Author: Anonymous | Category: N/A
Share Embed Donate


Short Description

Download KM - StrengthenKM - Strengthen and Weaken - PDF and Weaken...

Description

 

Stre n gth e n/Weo ke

n

Qu esti o ns

Strengthen/UUea ken Questions Directions: The questions in this section are based on the reasoning contained in brief statements or passages. For some questions, more than one of the choices could conceivably answer the question, However, you are to choose the best answer; that is, the response that most accurately and completely answers the question. You should not make assumptions that are by commonsense standards implausible, superfluous, or incompatible with the passage. L

Companypresident: Grievance procedures should

allow the grievant and the respondent to select a mediator who will attempt to work out a resolution. Grievances are costly and mediation could help to resolve many of them. However, beginning mediation fairly late in the process, as our human resources department proposes, would be relatively ineffective.

Which one of the following, if true, most helps to justify the company president's criticism of the human resources department's proposal?

(A)

(B) (C) (D) (E)

People who file grievances are unreasonabie

and would resist listening to a mediator. Many disagreements are already being solved

without the intervention of a mediator. Adversaries'positions tend to harden as a dispute wears on, making compromise less likely. Respondents tend to be supervisors who cannot give in to employees without losing authority. The mediation process itself is likely to cost as much in time and money as the present grievance procedures.

2.

v*lill

Advertisement: At most jewelry stores, the person assessing the diamond is the person selling it, so you can see why an assessor might say that a diamond is of higher quality than it really is. But because all diamonds sold at Gem World are certified in writing, you're assured of a fair price when purchasing a diamond from Gem World.

x,

In response to office workers'worries about the health risks associated with using video display

terminals (VDTs), researchers asked office rvorkers ro estimate both the amount of time they had spent using VDTs and how often they had suffered headaches over the previous year. According to the survey, frequent VDT users suffered from headaches more often than other office workers did, leading researchers to conclude that VDTs cause headaches. Which one of the following, if true, most undermin.r he researchers' conclusion?

(A)

ofthe offce workers surveyed participated in regular health programs during the year in

Few

question.

(B) (C) (D) (E)

fr

In their study the researchers failed to ask the workers to distinguish between severe migraine headaches and mild headaches. Previous studies have shown that the glare from VDT screens causes some users to suffer eyestrain. Office workers who experienced frequent headaches were more likely than other workers to overestimate how much time they spent using VDTs. Office workers who regularly used VDTs experienced the same amount of .iob-related stress as workers who did not use VDTs.

F

o E. t.i

g n o 0,

o o

=I =q

The reasoning in the advertisement would be most strengthened if rvhich one of the following were true?

(A)

(B) (C) (D)

(E)

Many jewelry stores other than Gem World also provide written certification of the quality of their diamonds. The certifications of diamonds at Gem World are written by people with years of experience in appraising gems. The diamonds sold at Gem World are generally ofhigher quality than those sold at other jewelry stores. The diamond market is so volatile that prices of the most expensive diamonds can change by hundreds of dollars from one day to the next. The written certifications of diamonds at G€m World are provided by an independent company of gem specialists.

Source: PrepTest 37, Sectlon 2, Queetlons

l,

6,

and

4

229

PDF compression OCR web optimization using a watermarked evaluation copy of CVISION PDFCompressor

 

4.

A plausible explanation ofthe disappearance ofthe dinosaurs is what is known as the comet theory. A large enough comet colliding with Earth could have caused a cloud of dust that enshrouded the planet and cooled the climate long enough to result in the dinosaurs' demise.

5. Nutritionisl Recently a caze has developed for

home juicers, $300 machines that separate the pulp of fruits and vegetables from the juice they contain. Outrageous claims are being made about the benefits of these devices: drinking the juice they produce is said to help one lose weight or acquire a clear complexion, to aid digestion, and even to prevent cancer. But there is no indication that juice separated from the putp of the fruit or vegetable has any properties that it does not have when unseparated. Save your money. If you want carrotjuice, eat a carrot.

Which one of the following statements, if true, most

UiO

seriously weakens the argument? (A) One of the various schools of paleontology adheres to an explanation for the disappearance of the dinosaurs that is significantly different from the comet theory. (B) Various species of animals f:rom the same era as the dinosaurs and similar to them in physiology and habitat did not become . extinct when the dinosaurs did. (C) It cannot be determined from a study of dinosaur skeletons whether the animals died from the effects of a dust cloud. (D) Many other animal species from the era of the dinosaurs did not become extinct at the same time the dinosaurs did. (E) The consequences for vegetation and animals

.= tr

of a comet colliding with Earth are not fully understood.

o ttl rE

trrlr

Which one of the following, if true, most calls into question the nutritionistt argument?

(A)

(B)

(c)

or vegetable. To,most people who would be tempted to buy a home juicer, 9300 would not be a major

(D)

The nutritionist was a member of a panel that extensively evaluated early prototypes of home iuicers. Vitamin pills that supposedly contain nutrients available elsewhere only in fruits and vegetables often contain a form of those compounds that cannot be as easily metabolized as the varieties found in fruits and vegetables.

(E)

tE

rg

u0

o

J

Most people find it much easier to consume a given quantity of nutrients in liquid form than to eat solid foods containing the same quantity of the same nutrients. Drinking juice from home juicers is less healthy than is eating fruits and vegetables because such juice does not contain the fiber that is eaten if one consumes the entire fruit

exPense.

Sources.' Prepiest

23O

'qry

37, Sectlon 4, Questlon t8

PrepTe* 38, Seetlon 7, Questlon 2

PDF compression OCR web optimization using a watermarked evaluation copy of CVISION PDFCompressor

 

gth en/Weo ke n Qu esti o ns Strength Stren Several companies will soon offer personalized electronic news services, delivered via cable or telephone lines and displayed on a television. People using these services can view continually updated stories on those topics for which they subscribe. Since these services will provide people with the information they are looking for more quickly and efficiently than printed newspapers can, newspaper sales

7

, Solicitor; Loux named Zembaty will. Her only beneficiary

includes the farmland in her estate. Thus, it is unlikely only that Loux would have had any objection to Zembaty's having sold it rather than having transferred it to her grandson.

will decline drastically if these services become

Which one of the following, if true, most seriously weakens the argument?

(B)

(c)

(D)

(E)

Which one of the following, if true, most weakens the solicitor's argument?

In reading newspapers, most people not only

look for stories on specific topics but also like to idly browse through headlines or pictures for amusing stories on unfamiliar or unusual

topics. Companies offering personalized electronic news services will differ greatly in what they charge for access to their services, depending on howwide a range oftopics they cover. Approximately 30 percent of people have never relied on newspapers for information but instead have always relied on nelvs programs broadcast on television and radio. The average monthly cost of subscribing to several channels on a personalized electronic news service will approximately equal the cost of a month's subscription to a newspaper. Most people who subscribe to personalized electronic news services will not have to pay extra costs for installation since the services will use connections installed by cable and telephone companies.

of

whom she was very fond. Prior to distributing the remainder to the beneficiary, Zembaty was legally required to choose which properties in the estate should be sold to clear the estate's healy debts. Loux never expressed any particular desire about the Stoke Farm, which

widely available.

(A)

executor of her r^ras her grandson,

(A) (B)

(C) (D) (E) 8.

The estate's debts could not have been cleared without selling the Stoke Farm. Loux repeatedly told her grandson that she would take care of him in her will. Louxwas well aware of the legal requirements the executor of her will would have to satisfy. The Stoke Farm was the main cause of the estate's debts. Loux's grandson had repeatedly expressed his desire to own a farm.

doctor: Sleep deprivation is the cause of many social ills, ranging from irritabfity to potentially dangerous instances of impaired decision making. Most people today suffer from sleep deprivation to some degree. Therefore we should restructure the workday to allow people flexibility in scheduling their work hours.

Medical

Which one of the following, if true, would most strengthen the medical doctor's argument?

(A) (B)

(C) (D) (E)

The primary cause of sleep deprivation is

overwork.

F q9. o f't 3.

n o

ql

tt

0

=. cq

Employees would get more sleep if theyhad greater latitude in scheduling their work hours. Individuals vary widely in the amount of sleep

they require. More people would suffer from sleep deprivation todaythan did in the past ifthe average number of hours worked per week had not decreased. The extent of one's sleep deprivation is proportional to the lengh of one's workday.

Source: PrepTest 36, Eectlon

3, Qucstlons 2, 5, and

7

PDF compression OCR web optimization using a watermarked evaluation copy of CVISION PDFCompressor

 

ISAT Mostery Practice 9.

@

10.

Peter:

Because the leaves of mildly drought-stressed plants are tougher in texture than the leaves of abundantly watered plants, insects prefer to feed on the leaves of abundantly watered plants. Therefore, to minimize crop damage,

.

farmers should water crops only just enough to ensure that there is no substantial threat, from a lack of water, to either the growth or

the yield of the crops. Jennifer: Indeed. In fact, a mildly drought-stressed

Alzheimer's disease might be caused by a virus. Which one of the following statements, if true, would most strengthen the scientist s hypothesis that Alzheimer's disease is caused by a virus?

plant will divert a small amount of its resources from normal growth to the development of pesticidal toxins, but abundantly watered plants will not.

(A) (B)

Which one of the following, if true, most strengthens Peter's argument?

(B)

a0 r-sE

o

9, l Itt

*

t

(c) (D)

(E)

IE

t,

tl

(C)

The leaves of some crop plants are much larger, and therefore absorb more water, than the leaves of some other crop plants. In industrialized nations there are more crops that are abundantly watered than there are crops grown under mild drought stress. Insect damage presents a greater threat to crop

(A)

plants than does mild drought stress. Farmers are not always able to control the amount of water that their crops receive when, for instance, there are rainstorms in the areas where their crops are growing. Mexican bean beetles are mor€ likely to feed on the leaves of slightly drought-stressed soybeans than oak lace bugs are to feed on the leaves of abundantly watered soybeans.

 g

o J.

A recent study suggests that Alzheimer's disease, which attacks the human brain, may be caused by a virus. In the study, blood from I 1 volunteers, each of whom had the disease, was injected into rats. The rats eventually exhibited symptoms of another degenerative neurological disorder, Creutzfeldt-f akob disease, which is caused by a virus. This led the scientist who conducted the study to conclude that

(D) (E) 11.

ffu -

Alzheimer's disease in rats is not caused by a virus. Creutzfeldt-|akob disease affects only motor nerves in rats'limbs, not their brains. The virus that causes Creutzfeldt-Jakob disease in rats has no effect on humans, The symptoms known, respectively, as Creutzfeldt-Iakob disease and Alzheimer's disease are different manifestations of the same disease.

Blood from rats with Creutzfeldt-|akob disease produced symptoms no other expetimental of the disease injected into rats. when

Marmosets are the only primates other than humans known to display a preference for using one hand rather than the other. Significantly more marmosets are left-handed than are right-handed. Since infant marmosets engage in much irnitative behavior, researchers hypothesize that it is by imitation that infant marmosets learn which hand to use, so that offspring reared by left-handed parents generally share their parents' handedness.

Which one of the following, if true, most supports the researchers' hypothesis?

(A)

(B) (C) (D) (E)

A study conducted on adult marmosets revealed that many were right-handed. Right-handed marmosets virtually all have at least one sibling who is left-handed. According to the study,33 percent of marmosets are ambidextrous, showing equal facility using either their left hand or their right hand. Ninety percent of humans are right-handed, but those who are left-handed are likely to have at least one left-handed parent. Marmosets raised in captivity with righthanded adult marmosets to whom thev are not related are more likely to be right-iranded than left-handed.

9ourcee: Preplest 36,

Prcpl$t 35,

S*tlon 3, Questlon 73, Sectlon

Preplest 33, Sectton

232

4, Questlon t0

L

Queetlon 7

I(APLAN

PDF compression OCR web optimization using a watermarked evaluation copy of CVISION PDFCompressor

 

Str e n gth

12. |uan:

Unlike the ancient Olympic games on which they are based, the modern Olympics include professional as well as amateur athletes. But since amateurs rarely have the financial or material resources available to professionals, it is unlikely that the amateurs will ever offer a serious challenge to professionals in those Olympic events in which amateurs compete

t

3.

against professionals' Hence, the presence professional athletes violates the spirit of of fairness essential to the games.

Which one of the following, if true, most strengthens the argument?

(A)

professionals should be allowed to compete.

Which one of the following, if true, most seriously undermines luaris argument?

(C) (D)

(E)

A recent national study of the trash discarded in several representative areas confirmed that plastics constitute a smaller proportion of all trash than paper products do, whether the trash is measured by weight or by volume. The damage that a given weight or volume of trash does to the environment is roughly the same whether the trash consists of plastics or paper products. Contrary to popular that of paper products.

is to showcase the rvorld's finest athletes, regardless of their backgrounds or resources. Hence,

(B)

(B)

In general, amateur athletes tend to

outnumber professional athletes in the modern OIYmPics. In certain events in the modern Olympics the best few competitors are amateurs; in certain other events the best few competitors are professionals. The concept of "amateur" and "professional" athletics would have been unfamiliar to the ancient Greeks on whose games the modern Olympics are based. In the modern Olympics there has been no noticeable correlation between the financial or material resources expended on the training of individual athletes and the eventual performance of those athletes. Many amateur athletes who take part in international competitions receive no financial or material support from the governments of the countries that the amateurs represent.

n/llleo ke n Quesfions

theenvironment plastics actually opinion, current use of does less therefore, nationwide harm to the than

Michiko: But the idea of the modern Olympics

(A)

e

(C) (D) (E) I4.

A given weight of paper product may increase in volume after manufacture and before being discarded as trash. Accordiag to popular opinion, volume is a more important consideration than weight in predicting the impact of a given quantity of trash on the environment. The sum of damage caused to the environment by paper trash and by plastic trash is greater than that caused by any other sort of trash that was studied. The production of any paper product is more harmful to the environment than is the production of an equal weight or volume of

A

)^(

any plastic. The proportion of plastic trash to paper trash varies from one part of the country to another.

There should be a greater use of gasohol. Gasohol is a mixture of alcohol and gasoline, and has a higher octane rating and fewer carbon monoxide emissions than straight gasoline. Burning gasohol adds no more carbon dioxide to the atmosphere than plants remol€ by photosynthesis.

F qg. o |rr g,

v o o tt o 3 II :l'

I

oe

Each of the following, if true, strengthens the argument above EXCEPT:

(A)

(B) (C) (D) (E)

Cars run less well on gasoline than they do on gasohol. Since less gasoline is needed with the use of gasohol, an energy shortage is less likely. Cars burn on the average slightly more gasohol per kilometer than they do gasoline. Gasohol is cheaper to produce and hence costs less at the pump than gasoline. Burning gasoline adds more carbon dioxide to the atmosphere than plants can remove.

Sources: PrepTest 33, Sectlon 7, Queetlons 4 and 6 PrcpTest

29, Sectlon 7, Questlon 3

KAPTA9 233

PDF compression OCR web optimization using a watermarked evaluation copy of CVISION PDFCompressor

 

ISAT Mostery

Prodice

Automobile-emission standards are enforced through

annual inspection. At those inspections cars are testid while idling; that is, standing still with their engines running. Testing devices measure the levels of various pollutants as exhaust gases leave the tail pipe.

16. Ianer

l'elevision programs and movies that depict violence among .eenagers are extremely popular. Given how influential these media are, we have good reason to believe that these depictions cause young people to engage in violent behavior. Hence, depictions -of violence among teenagers sliould be prohibited from movies and telev,ision p.og.u-i, if only in those programs and movies promoted to ' young audiences.

Which one of the following, if true, most strongly indicates that current enforcement of automobileemission standards might be ineffective in controlling overall pollutant levels?

(A) (B)

(C) (D)

bo

(E)

rIa) a ) tr -r

tl0

As an emission-control technology approaches its limits, any additional gains in effectiveness become progressively more expensive. The testing devices used must be recalibrated frequently to measure pollutant levels with acceptable accuracy. The adjustments needed to make a car idle cleanly make it likely that the car will emit high levels of pollutants when moving at

highway speeds. Most car owners ask their mechanics to make sure that their cars are in compliance with emission standards, When emission standards are set, no allowances are made for older cars.

Maurice-: But you are recommending nothing short of censorship Besides which, your claim that television and movie depictions of violence cause violence is mistaken: violence among young people predates movies and televisi6n

by centuries.

Which one of the following, if true, most strengthens fane's argument?

(A) (B)

(c)

o

a,

&

I

II

(D)

rg

I

bo

o J

(E)

The most violent characters depicted in movies and on television programs are adult characters who are portrayedly adult actors, The movies that have been'shorvn to have thc most influence on young people,s behavior are.those that are promoted to young audiences. The people who make the most profits in the movie and television industry ire those who can successfully promote their work to both young and old audiences. Many adolescents who engage in violent behavior had already displayed such behavior betore they lvere exposed to violence in movies. Among the producers who make both movies and television programs, many voluntarily restrict the subject matter of films directed toward young audiences.

l, euestion 24 Prcplest 29, Sectton g, euestion 6

Sources: Prcplest 29, Eection

234

PDF compression OCR web optimization using a watermarked evaluation copy of CVISION PDFCompressor

 

Stre n gth en/Wea ke

17.

In Yasukawa's month-long study of blackbirds, the percentage of smaller birds that survived the duration of the study exceeded the percentage of larger birds that survived. However, Yasukawa's conclusion that size is a determinant of a blackbird's chances of survival over a month-long period is probably misiaken, since smaller blackbirds are generally younger than larger ones. Which one of the following, if true, indicates that the

criticism ofYasukawa's research is based on misunderstanding of it?

(A) (B)

(C) (D)

(E)

a

Yasukarva compared the survival chances of two different species of blackbirds, a larger and a small species, rather than of different sizes of birds within one species, Yasukawa examined blackbirds in their natural

habitat rather than in captivity. Yasukawa did not compare the survival chances of blackbirds with those of other kinds of birds. Yasukawa noted that the larger blackbirds had more success in fights than did the smaller blackbirds.

blackbirds

Yasukawa noted that the larger tended to have more firmly established social

hierarchies than did the smaller blackbirds,

18.

n

Qu esti an s

Advances in photocopying technology allow criminals with no printing expertise to counterfeit paper currency. C)ne standard anticounter{eiting technique, microprinting, prints paper currency with tiny designs that cannot be photocopied distinctly. Although counterfeits of microprinted currency can be detected easily by experts, such counterfeits oftea circslate widely before being detected. An alternative, though more costly, printing technique would print currency rvith a special irk. Currency printed with the ink would change color depending on how ordinary light strikes it, whereas photocopied counterfeits of such currency would not. Because this technique would allow anyone to detect photocopied counterfeit currency easiill it should be adopted instead of microprinting despite the exPense.

Which one of the following, if true, most seriously undermines the argument?

(A)

(B)

(c) (D)

(E)

The longer the interval between the time " the counterfeit bill passes into circulation and time the counterfeit is detected, the more difficult it is for law enforcement officials to apprehend the counterfeiter.

)k

Sophisticated counterfeiters could produce currency printed with the special inkbut cannot

oq o

duplicate microprinted currency exactly. Further advances in photocopying technology will dramatically increase the level of detail that photocopies can reproduce. The largest quantities of counterfeit currency now entering circulation are produced by ordinary criminals who engage in

counterfeiting only briefly.

It is very difficult to make accurate estimates of what the costs to society would be if large amor.rnts of counterfeit currenry circulated widely.

F

n' g

7 o

ET

0

UT

 t tr 5

oq

Sources; PtepTest 27,Section 7, question 4 PrepTest 25, Sectlon 2, Question 5

I(APLAN

235

PDF compression OCR web optimization using a watermarked evaluation copy of CVISION PDFCompressor

 

ISAT Mastery Practice

19.

The Board of Trustees of the Federici Art Museum has decided to sell some works from its collection in order to raise the funds necessary to refurbish its galleries. Although this may seem like a drastic remedy, the curator has long maintained that among the paintings that the late Ms' Federici collected for the museum were several unsuccessful immature works by Renoir and C€zanne that should be sold because they are of inferior quality and so add nothing to the overall quality of the museum's collection. Hence, the board's action will not detract from the quality of the museum's collection.

20.

Each of the following,

argument EXCEPT:

(A)

Which one of the following, if true, most weakens the argument?

(A)

(B)

bo

.E tr

(c)

tu

D)

o a {g

&

tl

rtr

IJ

'f0 o J

(E)

(B)

The directors of an art museum can generally raise firnds for refurbishing the building in rvhich the museurt's collection is housed by means other than selling part of its collection. I'he quality of an art collection is determined not just by the quality of its paintings, but by *-hat its collection demonstrates about the development of the artistic talent and ideas of the artists represented. The immature works by Renoir and C€zanne that were purchased by Ms. Federici \i/ere at that time thought by some critics to be unimportant juvenile works. Those people who speculate in art by purchasing artworks merely to sell them at much higher prices welcome inflation in the art market, but curators of art museums regret the inflation in the art market. The best work of a great artist demands much higher prices in the art market than the rvorst work of that same artist.

A neighborhood group plans to protest the closing of the neighborhood's only recreation center on the grounds that to do so would leave the neighborhood without local access to a recreation center. "Our neighborhood already has the most residents per center of any neighborhood in the city," complained one resident, "and closing this center would make the situation unacceptable since access to recreational facilities is a necessity for this neighborhood."

(C) (D)

(E) 21.

if true, weakens the resident's

A large number of the neighborhood's residents are unable to travel outside their locality to gain access to recreational facilities. Children, the main users of recreational facilities, make up a disproportionately small segment of the neighborhood's population. Often the recreation center in the neighborhood is open but not being used. Programs that are routinely filled at other recreation centers must be canceled at the neighborhood's recreation center due to lack of interest. As people become computers morerecreation involved in games, and centers computer are

becoming increasingly less important.

Most small children are flat-footed. This failure of the foot to assume its natural arch, if it persists past early childhood can sometimes result in discomfort and even pain later in life. Traditionalll flat-footedness in children has been treated by having the children wear special shoes that give extra support to the foot, in order to foster the development of the arch.

Which one of the follorving, if true, most calls into question the efficacy of the traditional treatment described above?

(A)

(B) (C) (D) (E)

Many small children who have normal feet wear the same special shoes as those worn by flat-footed children. Studies of flat-footed adults show that flat feet are subject to fewer stress fractures than are feet with unusually high arches. Although most children s flat-footedness is corrected by the time the children reach puberty, some people remain flat-footed for life. Flat-footed children who do not wear the special shoes are as likely to develop natural arches as are flat-footed children who lyear

the special shoes. Some children who are not flat-footed have hip and lower leg bones that are rotated excessively

either inward or outward.

24, Sectlon 2, Questlon 70 PrepTest 22, Sectlon 2, Questlan 4

Sources: PrepTesl 25, Eection 4, Questlon PrepTest

236

I(APLAN

PDF compression OCR web optimization using a watermarked evaluation copy of CVISION PDFCompressor

 

Stre n gth en/We o ken Questi o n s

22.

activist: The closing of the nuclear power plant is a victory for the antinuclear cause. It also represents a belated acknowledgment by the power industry that they cannot operate such plants safely.

Antinuclear

Nuclear power plant manager: It represents no such thing. The availability of cheap power

from nonnuclear sources, together with the cost of mandated safety inspections and safety

repairs, made continued operation uneconomic. Thus it was not safety considerations but economic considerations that dictated the plant's closing.

Which one of the followiag, if true, most strongly supports the activist's claim of victory?

(A) B)

(c)

(D) (E)

The plant had reached the age at which its operating license expired. The mandate for inspections and repairs mentioned by the manager was recently enacted as a result of pressure ftom antinuclear groups. The plant would not have closed if cheap power from nonnuclear sources had not been available. Per unit of electricity produced, the plant had the highest operating costs of any nuclear power plant. The plant that closed had been able to provide backup power to an electrical network"rhen parts of the network became overloaded.

23.

A number of seriously interested amateur astronomers have tested the new Exodus refractor telescope, With it, they were able to observe in crisp detail planetaryfeatures that were seen only as{uzzy images in their 8-inch (approximately 20-centimeter) Newtonian telescopes, even though the 8-inch telescopes, with their wider apertures, gather more light than the 4-inch (approximately l0-centimeter) Exodus. Given these amateur astronomers' observational findings, any serious amateur astronomer ought to choose the Exodus if she or he is buying a telescope for planetary observation.

Which one of the following most seriously weakens the argument?

(A) (B)

Telescopes of certain types will not perform well unless they have been precisely collimated, a delicate adjustment requiring deftness. Image quality is only one of several different

factors that, taken together, should determine the choice of a telescope for planetary

(C) (D)

(E)

observation. Many serious amateur astronomers have of buying a telescope for intention observation.

no

F

planetaryotl O

The comparisons made by the amateur astronomers were based on observations made during several different observation sessions. The substance used to make the lenses of Exodus telescopes differs from that used in the lenses of other telescopes.

ft ql

-r

:-

n o

EI

g Ut

3 = -oc

Sourees: PrepTest 2 , Sectlan PrepTest 27, Section

2, Questlon 23 3, Questlan 2 I(A PLAN

237

PDF compression OCR web optimization using a watermarked evaluation copy of CVISION PDFCompressor

 

24.

A director of the Rem Pharmaceutical Company argued that the development costs for new vaccines that the health department has requested should be subsidized by the government, since the marketing of vaccines promised to be less profitable than the marketing of any other pharmaceutical product. In support of this claim the director argued that sales of vaccines are likely to be lower since each vaccine is administered to a patient only once, whereas medicines that combat diseases and chronic illnesses are administered many times to each patient.

26.

commercial vehicles. Clearly, triple-trailers are safer

than other commercial vehicles.

Which one of the following, if true, most substantially weakens the argument? {A) It takes two smaller semitrailers to haul as much weight as a single triple-trailer can. (B) Ilighways in the sparsely populated West are much less heavily traveled and consequently are far safer than highways in the national

Which one of the following, if true, most weakens the support offered by the company director for the claim concerning the marketing of vaccines?

(A)

(B) (C) (D)

ua

g tr o U}

(E)

|lI

G'

o

&

Itr -f,

'to o J

25.

Vaccines are administered to many more people than are most other pharmaceutical products' Many of the diseases that vaccines are designed to prevent can be successfully

treated by medicines.

Pharmaceuticalcompaniesoccasionally market products that are neither medicines nor vaccines. Pharmaceutical companies other than the Repi Company produce vaccines. Pharmaceutical The cost of administering a vaccine is rarely borne by the pharmaceutical company that manufactures that vaccine,

will often develop a strong distaste for the one food in the meal that had the most distinctive flavor, rvhether or not Someone who gets sick from eating a meal

that food caused the sickness' This phenomenon explains why children are especially likely to develop strong aYersions to some foods. Which one of the following, if true, provides the strongest support for the explanation?

Opponents of allowing triple-trailer trucls to use the national highway system are wrong in claiming that these trucks are more dangerous than other commercial vehicles. In the western part of the country, in areas where triple-trailers are now permitted on some highlvays, for these vehicles the rate of road accident fatalities per mile of travel is lower than the national rate for other types of

Opponents of the triple-trailers also once

(D)

In

(E) 27.

system as a whole.

(C)

::1":;1*iliT',i:*l.Ttff #"1:l*' areas where the

drivers need

triple-trailers are permitted,

a special license

For rate ofwas road accident triple-trailers per milethe of travel higher last fatalities year than in the two previous years,

A newly developed light bulb is much more costeffective than conventional light bulbs: it costs only about 3 times what a conventional light bulb costs but it lasts up to l0 times as long as a conventional light bulb. Despite the manufacturer's intense efforts to publicize the advantages of the new bulb, one analyst predicts that these new bultrs will prove to sell very poorly. Each of the following,

if true, provides support for

(A)

Children are more likely than adults to be given meals composed of foods lacking especially distinctive fl avors.

the analyst's prediction EXCEPT:

(B)

Children are less likely than adults to see a connection between their health and the foods they eat. Children tend to have more acute taste and to become sick more often than adults do' Children typically recover more slorvly than adults do from sickness caused by food. Children are more likely than are adults to refuse to eat unfamiliar foods.

(B)

(C) (D) (E)

to operate them.

(A)

(C) (D) (E)

The light generated by the new bulb is in the yellow range of the spectrum, a type of

artificial light most people find unappealing. Most people who purchase light bulbs prefer to buy inexpensive light bulbs rather than more durable but expensive light bulbs. A manufactuter of one brand of conventional light bulb has advertised clairns that the new light bulb uses more electricity than do conventional iight buibs. The new bulb is to be marketed in several different quantities, ranging frorn packages containing one bulb to packages containing four bulbs.

A competing manufacturer is about to introduce a light bulb that lasts 10 times as long as a conventional bulb but costs less than a

conventional bulb.

Sources: PrepTest 20, Section 7, Questions 2 and g PrepTest 2O, Sectlon 4, Questlons 3 and 5

238 PDF compression OCR web optimization using a watermarked evaluation copy of CVISION PDFCompressor

 

Str e n gth 28.

For every 50 dogs that contract a certain disease, one will die from it, A vaccine exists that is virtually 100 percent effective in preventing this disease. Since the iisk of death from complications of vaccination is one death per 5,000 vaccinations, it is therefore safer for a dog to receive the vaccine than not to receive it.

Which one of the following would it be most helpful to know in order to evaluate the argument? (A) the total number of dogs that die each year from all causes taken together (B) whether the vaccine is effective against the disease in household pets other than dogs (C) the number of dogs that die each year from diseases other than the disease in question tD) the likelihood that a dog will contract another

(E) 29.

disease such as rabies the likelihood that an unvaccinated dog contract the disease in question

will

Public health will improve more quickly in the wake of new medical discoveries if medical researchers abandon their practice of waiting until their findings are published in peer-reviewed journals before

informing the press of important research results, This is because the public release of new medical information allows people to use that information in order to improve their health, but the peer-review process is unavoidablY verY slow

30.

(A)

(B) (C) (D) (E)

Peer review often prevents the publication of false conclusions drawn on the basis of poorly conducted medical research. People often alter their life-styles on the basis of nerv medical information made available

n/ We o ke n

Qu esti o n s

Three major laundry detergent manutacturers have concentrated their powdered detergents by reducing the proportion of inactive ingredients in the detergeirt formulas. The concentrated detergents will be sold irr smaller packages. In explaining the change, the manufacturers cited the desire to reduce cardboard packaging and other production costs. Market analysts predict that the decision of these three manufacturers, who control 80 percent of the laundry detergent market, will eventually bring about the virtual disappearance of old-style bulky detergents.

Which one of the following, if true, most strongly

supports the prediction made by the market analysts?

(A)

(B)

(c)

(D)

Which one of the following, if true, most seriously weakens the argument?

e

(E)

Most smaller manufacturers of laundry detergents will consider it too expensive to retool factories for the production of the

smaller detergent packages. Many consumers will be skeptical initially that n the recommended small amount of tk concentrated detergent will clean laundry as effectively as the larger amount of the oldstyle detergent did. Some analysts believe that consumers will have a greater cost per load of laundry to to pay use the new concentrated detergent than they did to use the old-style detergent. Major supermarkets have announced that they will not charge the detergent nranufacturers less to display their detergents, even though the detergents will take up less shelf space. Consumers are increasingly being persuaded by environmental concerns to buy concentrated detergents when available in order to reduce cardboard waste'

r

oe €

-t rt $ r

fr o Ot

{rl

() r = 5

oe

through the Press. Some improvements in public health are due to factors other than the discovery of new

information.

medical newspapers would be willing to publish Some the results of medical research before those results have appeared in peer-reviewed

journals. Most peer-reviewed scientific journals would refuse to give up the practice of peer review.

Sources.' PrepTest 20, Sectlon PrepTest 17, Sectlon PrcpTest 77, Sectlon

4, Queetlon 7 2, Queslion x'3 3, Questlon I I(APLAN

239

PDF compression OCR web optimization using a watermarked evaluation copy of CVISION PDFCompressor

 

ISAT Mastery Proctice Ouestions 31.-32

Although tales of wonder and the fantastic are iategral to all world literatures, only recently has the fantasy genre

had a comrnercial resurgence in North America. During the last 20 years, sales of fantasy-fiction books written for adults have gone from 1 to l0 percent of total adult-fiction sales. At the same time, the number of favorable reviews fantasy books has increased markedly. Some booksellers of say that the increased sales of fantasy books written for adults can be traced to the increased favorable attention given the genre by book reviewers. 3

l.

Which one of the following, if true, undermines the booksellers' explanation of the growth in sales of fantasy-fiction books for adults?

(A)

(B) ba ? -rr I-

(C)

g C) a

(D)

E

(E)

G

{t

T

G

I b'0 o J

II

32.

Publishers often select a manuscript on the basis of whether they think that the published book will receive favorable reviews by trook reviewers. Few readers of fantasy fiction read book reviews, and even fewer select books to purchase on the basis of those reviews. are aware of about Most what maior reviewers have bookbooksellers written reiently published books.

Although the increase in the percentage of

33.

@

Advertisement: Most power hedge trimmers on the market do an adequate job of trimming hedges, but many power hedge trimmers are dangerous to operate and can cause serious injury when used by untrained operators. Bolter Industries'hedge trimmer has been tested by National Labotatories, the most

name in safety testing. So you know, trusted you buy a Bolter's, you are buying a power hedge trimmer whose safety is assured.

if

The answer to which one of the following questions would be most usefiil in evaluating the truth of the conclusion drawn in the advertisement?

(A)

Has National Laboratories performed safefy tests on other machines made by Bolter

(B)

How important to the average buyer of a power hedge trimrner is safety of operation? What were the results of National Laboratories' tests of Bolter Industriest hedge trimmer? fue there safer ways of trimming a hedge than using a power hedge trimmer? Does any other power hedge trimmer on the market do a better job of trimming hedges than does Bolter Industries'hedge trimmer?

(c) (D) (E)

Industries?

fantasy books sold has been substantial, publishers estimate that sales could increase even further, Many of the book reviews of new fantasyfiction novels also mention great fantasy novels ofthe past.

Which one of the following, if true, most strongly supports the booksellers'explanation of the growth

in

sales

(A)

(B) iC) (D)

(E)

offantasy-fiction books for adults?

Many experts report that on average the reading level of book buyers has declined over past years. 20 during the past 20 years has the life Because become complex and difficult, many readers

have come to prefer the happy endings that fantasy fiction often provides. Some fantasy publishers take advantage of the popularity of certain books by commissioning similar books. Because few readers of mystery novels were buyrng fantasy fiction, 10 years ago the major publishers of fantasy fiction created an advertising campaign directed specifically at those readers. After fantasy fiction began to be favorably reviewed by respected critics 20 years ago, book buyers began to regard fantasy books as suitable reading for adults. Sorrrces; PrepTest 77, Sectlon 3, euesfjons 7 and PrepTest 7O, Section 7, Question 6

lg

240 PDF compression OCR web optimization using a watermarked evaluation copy of CVISION PDFCompressor

 

Ouestions 34-35

36.

A physician who is too thorough in conducting a medical checkup is likely to subject the patient to the discomfort and expense of unnecessary tests. One who is not thorough enough is likely to miss some serious problem and therefore give the patient a false sense of security. It is difficult for physicians to judge exactly how

The foreign minister of Zeria announced today that her country was severing diplomatic relations with Nandalo because of Nandalo's flagrant violations of human rights. But Zeria continues to maintain diplomatic relations with many countries that the minister knows to have far worse human-rights records than Nandalo does. Therefore, despite the

thorough they should be. Therefore, it is generally unwise for patients to have medical checkups when they do not

move foreign be minister's claim, this latest cannot explained exclusively by diplomatic Zeria's

feel

34,

ill.

Which one of the following, if true, would most seriously weaken the argument in the passage? (A)

(B)

(C) (D) (E) J).

Some serious diseases in their early stages have symptoms that physicians can readily detect, although patients are not aware of any

problem. Under the Dressure of reduced

reimbursements, physicians have been reducing the average amount of time they spend on each medical checkup. Patients not medically trained are unable to judge for themselves what degree of thoroughness is appropriate for physicians in conducting medical checkups. Many people are financially unable to afford regular medical checkups. Some physicians sometimes exercise exactly the right degree of thoroughness in performing a medical checkup.

Which one of the following, if true, would provide the most support for the conclusion in the passage? (A)

Not all medical tests entail significant

(B)

Sometimes, unnecessary medical tests cause healthy people to become ill. Some patients refuse to accept a physician's assurance that the patient is healthy.

(c) (D)

(E)

commitment to upholding human rights.

Which one of the follorving, if true, provides the most support for the argument in the passage? (A)

(B)

(c)

(D) (E)

The country that currently buys most of Zeria's exports recently suggested that it might severely restrict its imports fromZeria unless Zeria broke off diplomatic relations with Nandalo. Two weeks after the Zeian minister's announcement, several other countries cited human-rights violations as a reason for severing diplomatic relations with Nandalo. More countries have expressed concern over

reported human-rights violations in Nandalo than have expressed concern over human-rights violations in Zeria. Nandalo has considered accusing Zefia of violating the human rights of Nandalo citizens living il Zeria. The opposition party rn Zeia has long advocated severing trade relations with countries that systematically violate human rights but has opposed severing diplomatic

relations.

F. qo fr gl ; o

AI

o

UT

=. oq

discomfort.

The more complete the series of tests performed in a medical checkup, the more likely it is that a rare disease, if present, will be discovered. Physicians can eliminate the need to order certain tests by carefully questioning patients and rejecting some possibi-lities on that basis.

Source: Prepfest 7O, Sectlon

4,

Questions

l,

2, and

tl ?41

PDF compression OCR web optimization using a watermarked evaluation copy of CVISION PDFCompressor

 

Mostery Prodice

LSAT

J7

lhe number of aircraft collisions on the ground is increasing because ofthe substantial increase in the number Jf ttigtttt operated bv the airlines. Many of the tatalittes that occur in such collisions are caused not by the collision itself, but by an inherent flaw in

39.

Policy

basic human right;

ideas are openly aired, good ideas flourish, silly proposals are easily recognized as such, and dangerous ideas can be responded to by rational argunrent. Nothing is ever gained by

forcing citizens to disseminate their thoughts

of fatalities Therefore-, annually reduce the to number total from such collisions, the airlines that result should be required to remove all seats that restrict access to emergencY exits.

g o UI

(E)

rg

o g, -tg

IJ ao

rrr

o

J

in secret.

Which one of the following, if true, would most strengthen the argument?

(A)

Which one of the following, if true, provides the most suppori for the ProPosal? (A) The number of deaths that occurred in theater fires because theater Patrons could not escape was greatly reduced when theaters were required to have aisles leading to each exit' (B) Removing the seats that block emergency exits on aircraft rvill require a costly refitting of aircraft cabins. (C) In the event of fire, public buildings equipped with smoke detectors have fewer fatalities than do public buildings not so equipped'

(D)

38.

In the event of collision, passengets on planes rvith a smaller passenger capacity generally suffer more serious injury than do passengers on planes with a larger pass€nger capacity. The safetybelts attached to aircraft seats function to Protect passengers from the full force of impact in the event of a collision.

When a study of aspirin s ability to Prevent heart attacks in humans yielded positive results' researchers immediately submitted those results to a medical journal, which published them six weeks later. Had the results been published sooner, many of the heart attacks that occurred during the delay could have been Prevented.

The conclusion drawn above lvould be most undermined if it were true that

(A)

(B) (C) (D) (E)

the medical journal's staff worked overtime in order to publish the study's results as soon as possible studies of aspirin's usefulness in reducing heart attacks in laboratory animals remain inconclusive people who take aspirin regularly suffer a higher-than-average incidence of stomach ulcers the medical journalt officiai policy is to publish articles only after atr extensive review process a person's risk of suffering a heart attack drops only after that person has taken aspirin

it

speech is not only a is also the only rational

policy for this government to adopt. When

the cabin design of most aircraft, in which seats, by restricting access to emergency exits, impede escape'

blo C rn

Adviser: Freedom of

(B) (C) (D) (E)

Most citizens would tolerate some limits on fteedom of speech. With or without a poliry of freedom of speech, governments respond to dangerous ideas

irrationally.

Freedom of religion and freedom of assembly are also basic human rights that governments must recognize, Governments are less likely to be overthrown if they openly adopt a policy allowing freedom of speech. Great ideas have flourished in societies that repress free speech as often as in those that

permit it.

40.

The use of money causes a civilization to decline. That this is true is shown by the way the troubles of Western civilization began with the invention of money. While real money (gold and sil'r'er) is bad enough, imitation money (paper money) is a horror.

The decline of Western civilization exactly parallels the increasing use of money-both real money and worthless paper money-as a substitute for things of intrinsic value.

Which one of the following, if true, could contribute most to a refutation of the argument?

(A)

(B) ((,) (D) (E)

People prefer using lnoney to having a system in *'hich goods are bartered for other goods

of equal intrinsic value.

Eastern cultures have used money' and Eastern civilizations have not declined. 'Ihe use of paper money encourages disregard tbr the value of work because the money itself has no intrinsic value. The rate of exchange befiveen gold and paper money has tluctuated greatiy in Western

civilization.

Some employers exchange goods for their

employees'services in order to avoid the exchange of money.

regularly for two Years

Sources,' PrepTest 9, Section PrepTest 9, Section PrepTest 6, Section Preplest 6, Sectlon

) A)

2, I 4, Questlon Questlan 2 2, Questlon 21 3, Questlon 2

I(A PLAN)

PDF compression--zOCR web optimization using a watermarked evaluation copy of CVISION PDFCompressor

 

Stre n gth e n/We

4t. In an attempt to counter complaints that a certain

pesticide is potentially hazardous to humans if absorbed into edible plants, the pesticide manufacturer has advertised that "ounce for ounce, the active ingredient in this pesticide is less toxic than the active ingredient in mouthwash."

Which one of the following, if true, indicates

a

weakness in the manufacturer's argument?

(A)

(B) (C) (D) (E)

42.

The ounce-for-ounce toxicity of the active ingredient in mouthwash is less than that of most products meant for external use by humans, such as nail polish or other cosmetics' The quantity of toxins humans ingest by consuming plants treated with the pesticide is, on average, much higher than the quantity of toxins humans ingest by using mouthwash. The container in which the pesticide is packaged clearly identifies the toxic ingredients and carries warnings about their potential danger to humans. On average, the toxins Present in the pesticide take longer than the toxins present in mouthwash to reach harmful levels in the

43.

Recently, highly skilled workers in Eastern Europe have left jobs in record numbers to emigrate to the West. It is therefore likely that skilled workers who remain in Eastern Europe are il high demand in their home countries.

n Quesfrons

Sedimentary rock hardens within the eartir"s crust as layers of matter accumulate and the pressure of the layers above converts the layers below into rock, One particular layer of sedimentary rock that contains an unusual amount of the element iridium has been presented as support for a theory that a meteorite collided with the earth some sixty million years ago. Meteorites are rich in compared the iridium earth's crust, and geologists theorize that to a meteorite's collision with the earth raised a huge cloud of iridium-laden dust. The dust, they say, eventually settled to earth where it combined with other matter, and as new layers accumulated above it, it formed a layer of iridium-rich rock.

Which one of the following, if true, would counter the claim that the iridium-rich layer described in the passage is evidence for the meteorite collision theory? (A) (B)

(c)

human body.

Since the government began to regulate the pesticide industry over ten years ago, there has been a growing awareness ofthe dangers of toxins used in pesticides.

a ke

(D)

(E)

The huge dust cloud described in the passage would have blocked the transmission of sunlight and lowered the earth's temperature. A layer of sedimentary rock takes millions of years to harden. Layers of sedimentary rock are used to

determine the dates of prehistoric events rvhether or not they contain iridium. Sixty million years ago there was a surge in volcanic activity in which the matter spewed from the volcanoes formed huge iridium-rich dust clouds. The iridium deposit occurred at about the same time that many animal species became extinct and some scientists have theorized that mass dinosaur extinctions were caused by a meteorite collision.

(C) (D) (E)

fr o

gr Yl

c

oe

weakens the argument?

(B)

oT 'Q fr o rt

I = )II

Which one of the following, if true, most seriously

(A)

F

Eastern European factories prefer to hire workers from their home countries rather abroad. than to importinworkers economic Easternfrom European changes Major structures have led to the elimination of many positions previously held by the highly

skilled emigrants. Many Eastern European emigrants need to acquire new skills after finding rvork in the West.

Eastern European coutrtries plan to train many nerv rvorkers to replace the highly skilled workers lvho have emigrated. Because of the departure of skilled lvorkers from Eastern European countries, many positions are now unfilled.

Sourcesi Pseplest Q Sectfon 3, Questlon 4 PrepTest 5, Sectlon 7, Questions 5 and

7 243

PDF compression OCR web optimization using a watermarked evaluation copy of CVISION PDFCompressor

 

44.

essence, all rent-control policies involve specifrilg maximum rent that a landlord may charge for a dwelling. The rationale for controlling rents is to protect tenants in situations where limited supply will cause rents to rise sharply in the face of increased demand, However, although rent control may help some tenants in the short run, it affects the rental-housing market adversely in the long run because landlords become reluctant to maintain the quality of their existing properties and even more reluctant to have additional rental-housing units

In a

built.

Which one of the following, if true, best explains the landlords' reluctance described above?

(A)

(B)

c) GO

g t: o Q

{t-:

{tt

(D) (E)

{D

*

fl

Tenantspreferlow-qualityaccommodations with rent control to high-quality accommodations without it. Rent control makes it very difficult for

landlords to achieve reasonable returns on any investments in maintenance or in new construction. Rent control is a common practice even though it does nothing to alleviate shortages

in rental housing. Rent control is generally introduced for politicai reasons and it takes political action to have it lifted again. Tenants prefer rent control to the alternative of receiviag direct government subsidies toward rents they cannot afford.

45.

Several studies have shown that hospitals are not all equally successfi.rl: patients are much more likely to die in some of them than in others. Since the

hospitals in the studies had approximately equal per-patient funding, differences in the quality of care provided by hospital staff are probably responsible

for the differences in mortality rates.

Which one of the following, if true, casts the most doubt on the conclusion drawn aboye?

(A)

(B) (c) (D) (E)

The staff in some of the hospitals studied had earned more advanced degrees, on average, than the staff in the other hospitals. Patient populations vary substantially in average severity of illness from hospital to

hospital.

The average number of years that staff members stay on at a given job varies considerably from one hospital to another. Approximately the same surgical procedures were performed in each of the hospitals covered in the studies. Mortality rates for hospital patients do not vary considerably from one region of t}re

country to another.

|r

frl r: b0

o

-J

Soutc6s.'PrepTest 5,9ectlon PrepTest 5, Sectlon

l,

Questlon 29

3, Question 2

244 PDF compression OCR web optimization using a watermarked evaluation copy of CVISION PDFCompressor

 

Str e n gth 46.

Compared to us, people who lived a century ago had very few diversions to amuse them. Therefore, they likely read much more than we do today.

48. Lydia:

weakens the arprment?

(B) (C) (D) (E) 47.

Many of the books published a century ago quality. r+'ere of lowpeople literary who lived a century ago On average, had considerably less leisure time than we do

today. The number of books sold today is larger than it was a century ago. On the average, books today cost slightly less in relation to other goods than they did a century ago. One of the popular diversions of a century ago was horse racing.

The vomeronasal organ (VNO) is fourrd inside the noses of various animals. \{hile its structural development and function are clearer in other animals, most humans have a Vl.{O that is detectable, though only microscopically. \Vhen researchers have been able to stimulate VNO cells il hurnans, the subjects have reported experiencing subtle smell sensations. It seems, then, that the VNO, though not completely understood, is a functioning sensory organ in most hurnans.

Which one of the follorving, if true, most weakens the argument?

(A)

(B) (C) (D) tE)

It is not known whether the researchers succeeded in stinrulating only VNO cells in the human subiects' noses. Relative to its occurrence in certain other animals, the human VNC) appears to be anatomically rudimentary and underdeveloped. Certain chemicals that play a leading role in

n

Qu esfions

Red squirreis are knon'n to make holes in

the bark of sugar maple trees and to consume the trees'sap. Since sugar maple sap is essentially rvater with a small concentration of sugar, the squirrels almost certainly are after either water or sugar. Water is easily available from other sources in places where maple trees grow, so the squirrels u'ould not go to the trouble of chewing holes in trees just to get water. Therefore, they are probably after the sugar.

Which one of the followi,ng statements, if true, most

(A)

e n /VVe a ke

Galina: It

must be something other than sugar, because the concentration of sugar in the maple sap is so low that a squirrel would need to drink an enormous amount of sap to get any significant amount of sugar.

Which one of the following, if true, rrrost undermines the force of Galina's attempted rebuttal of Lydia's argument{

(A) (B)

(C) (D) (E)

klo*'n to like foods that have a high concentration of sugar.

s9 ?

Squirrels are

Once a hole in a sugar maple trunk has provided one red squirrel rvith sap, other red squirrels will make additional holes in its

trunk.

Trees other than sugar maples, whose sap contains a lower concentration of sugar than does sugar mapie sap, are less frequently

tapped by red squirrels. Red squiruels leave the sugar maple sap that slowly oozes out of the holes in the tree's trunli until much of the water in the sap has evaporated. During the season rvhen sap can be obtained fiom sugar maple trees, the weather often becomes cold enough to prevent sap from oozing out of the trees.

F

o oq tl f1

 t .-

n o gl

U}

o I

=q

-r

the way the VNO functions in animals in which it is highly developed do not appear to play a role i-n its functioning in humans.

Secondaryanatomicalstructuresassociated with the VNO in other animals seem to be

absent in humans. For many animal species, the VNO is thought to subtly enhance the sense of snell.

$ources: PrepTest 37, Seation 2, Question 4 PrcpTest 37, Sectlon

tl r(APTAN' ')d1 \-.-l

4, Questlons 2 and

PDF compression OCR web optimization using a watermarked evaluation copy of CVISION PDFCompressor

 

ISAT Mastery 49,

Pradice

When several of a dermatologist's patients complained of a rash on just one side of their faces, the dermatologist suspected that the cause was some kind of external contact. In each case it turned out that the rash occurred on the side of the face to which the telephone was held. The dermatologist concluded that the rash was caused by prolonged contact with telephones.

Each of the following,

if true, provides additional

support for the dermatologist's diagnosis EXCEPT:

(A) (B)

(C)

)^(

*

(D) (E)

ba

ttt -

s -r

o

Ill {g (u

&

-IE l:

ta a0

o

-l

50.

Many telephones are now manufactured using a kind of plastic to which some people are

mildly allergic.

Contact between other devices and the patients'faces occurred equally on both sides

of their faces. Most of the patients had occupations that required them to use their telephones

extensively. Telephones are used by most people in the

industrialized world. The complaints occurued after an increase in the patients' use of the telephone.

In ancient Mesopotamia, prior to 2900 B.C., wheat was cultivated in considerable quantities, but after 2900 B.C. production of that grain began to decline as the production of barley increased sharply. Some

historians who study ancient Mesopotamia contend that the decline in wheat production was due to excessive irrigation, lack ofdrainage, and the consequent accumulation of salt residues in the soil.

51.

It has been claimed that television networks shoutd provide equal time for the presentation of opposing views whenever a television program concerns scientific issues*such as those raised by the claims of environmentalists-about which people disagree. However, although an obligation to provide equil time does arise in the case of any program concerning issues, it does so because social social issues almost always have important political implications and seldom can definitely be settled on the basis of available evidence. If a program concerns scientific issues, that program gives rise to no such equal time obligation. Which one of the following, if true, most seriously

weakens the argument?

(A) (B)

{C) (D) (E)

No scientific issues raised by the claims of environmentalists have important political implications. There are often more than two opposing views on an issue that cannot be definitely settled on the basis of avai_lable evidence. Some social issues could be definitely settled on the basis of evidence if the opposing sides would give all the available evidence a fair

hearing. Many scientific issues have important political implications and cannot be definitely settled on the basis of the available evidence. Some television networks refuse to broadcast programs on issues that have important political implications and that cannot be definitely settled by the available evidence.

Which one of the following, if true, most helps to support the historians' contention concerning the reasons for the decline in wheat production in ancient Mesopotamia?

(A)

(B) (C) (D) (E)

The cultivation of barley requires considerably less water than does the cultivation of wheat, Barley has much greater resistance to the presence of salt in soi_l than does wheat. Prior to 2900 8.C., barley was cultivated along with wheat, but the amount of barley produced was far less than the amount of lvheat produced, Around 2900 8.C., a series of wheat blishts occurred, destroying much of the whe-at crop year after year. Literary and archaeological evidence indicates that in the period following 2900 8.C., barley becarne the principal grain in the diet of most of the inhabitants of Mesopotamia.

Sources: PrepTest 37, Sectlon Prep7est 36, Sectlon

4, Questlons t3 and 25

l,

Question 8

246 PDF compression OCR web optimization using a watermarked evaluation copy of CVISION PDFCompressor

 

Stre n gth en/Wea ke n Qu esti o n s 52.

Journal: In several ps1'chological studies' subjects

54,

rvere given statements to read that caused

them to form ner,r'beliefs' Later, the subjects were told that the original statements lt/ere false, The studies report, however, that most subjects persevered in their newly acquired beliefs, even after being told that the original suggests false. This statements were strongly acquired onto hold to continue that humans beliefs even in the absence of any credible evidence to suPPort them.

Which one of the following statements, if true, most weakens the argument?

Which one of the following, if true, most undermines

(A)

the journal's argument?

(A)

(B) (C) (D) (E) f,J.

ofthe truth ofrvhat the subjects were later told, the beliefs based on the original statements were, for the most part,

Regardless

correct' It is unrealistic to expect people to keep track of the original basis of their beliefs, and to revise a belief when its original basis is undercut. The statements originally given to the subjects would be highly misleading even if true. Most of the subjects had acquired confirmation of their newly acquired beliefs by the time they were told that the original

conclusion drawn in the magazine's survel'. if true, would strengthen the statistician's argument EXCEPT:

Each of the following,

(C) (D) (E)

(D)

Insects build up resistance

(B)

(E) 55.

Expert

The credibility of the magazine has been called into question on a number of occasions' The conclusions drawn in most magazine surveys have eventually been disproved' Other surveys suggest that North Americans are just as concerned about politics as they are about finances. There is reason to be skeptical about the results of surveys that are biased and unrePresentative. Other surveys suggest that North Americans are concerned not only with politics and finances, but aiso with social issues.

to farm

crops.

* JL_ more readily to B.t. )^(

toxins than to chemical insecticides. Birds and rodents often do greater damage to farm crops than do insects.

F

o

witness: Ten times, and in controlled OE circumstances, a single drop of the defendant's And fabric. in fail onto the to blood was allowed all ten cases, the stained area was much less than AI the expected 9.5 cm2.In fact, the stained area was alwuys between 4.5 and 4.8 cm2. I conclude that a single drop of the defendant's blood stains gl much less than 9.5 cm/ of the fabric,

E'

statements were false.

Americans are more concerned about their personal finances than about politics. One question was: "Which do you think about more: politics or the joy of earning money?" This question is clearly biased' Also, the readers of ihe magazine are a self-selecting sample. Thus' there is reason to be skeptical about the

(B)

(c)

Chemical insecticides cause harm to a greater number of insect species than do B't. toxins. No particular B.t. toxin is effective against all insects, B.t. toxins do not harm weeds that do damage

Most of the subjects were initially skeptical of the statements originally given to them.

Statistician: A financial magazine claimed that its survey of its subscribers showed that North

(A)

One of the advantages of Bacillus thuringiensis (B't.) toxins over chemical insecticides results from their specificity for pest insects. The toxins have no known detrimental effects ott mammals or birds. In addition, the limited range of activity of the toxins toward insects means that often a particular toxin will kill pest species but not affect insects that prey upon the species. This advantage makes B't. toxins preferable to chemical insecticides for use as components of insect pest management programs.

-7

o

Which one of the following, if true, most undermines the value of the evidence for the expert witness's conclusion?

(A)

(B) (C) (D) (E)

If similar results had been found after 100 test drops of the defenclant's blood' the evidence

rvould be even stronger. Expert witnesses have sometimes been known to fudge their data to accord with the prosecution's case. In an eleventh test drop of the defendant's blood, the area stailed was also less than 9.5 cm2'-this time stainin g9.3 cmz. Another person's blood was substituted' and in otherwise identical circumstances, stained between 9.8 and 10.6 cmz of the fabric' Not all expert witnesses are the authorities in their fields that they claim to be.

Sources,' PrepTest 35, Sectlon 1, Questions 13 and PrepTest 35,9ection 4, Question 5 Prcplest 33, Section Question t7

t5

t,

247

(n

o 3 rtt 5

oq

PDF compression OCR web optimization using a watermarked evaluation copy of CVISION PDFCompressor  

ISAT Mostery Proctice

56.

Sales

manager: Last year the total number of meals sold in our company's restaurants was much higher than it was the year be{bre. Obviously consumers find our meals desirable.

Accountant: Ifyou look at individual restaurants, however, you find that the number of meals

sold actually decreased substantially at every one of our restaurants that was in operation both last year and the year before, The desirability of our meals to consumers has clearly decreased, given that this group of restaurants-the only ones for which we have sales figures that permit a comparison between last year and the year before-demonstrates a trend toward ferver sales.

Which one of the following, if true, most seriously calls into question the accountant's argument?

)^(

(A)

*

(B)

bo

rrIF

(c)

tr

o la o o

(D) (E)

&,

r

II

o I

00

o J

57,

The company's restaurants last year dropped from their menus most of the new dishes that had been introduced the year before. Prior to last year there was an overall downward trend in the company's sales. Those restaurants that large of the increase theircompany's sales last year did not offerdid discounts on prices to attract customers. Sales of the company's most expensive meal contributed little to the overall two-year sales increase. Most of the company's restaurants that were in operation throughout both last year and the year before are located in areas where residents experienced a severe overall decline in income last year,

In defending the llyperion School of ]ournalism from charges that its program is of little or no value to its students, the dean of the school pointed to its recent success in placing students: 65 percent of its graduates rvent on to internships or jobs in print or

5b.

Advances in photocopying technology allow criminals with no printing expertise to counterfeit paper

currency. One standard anticounterfeiting technique, microprinting, prints paper currency with tiny designs

that cannot be photocopied distinctly. Although counterfeits of microprinted currency can be detected easily by experts, such counterfeits often circulate

widely before being detected. An alternative, though more costll', printing technique would print currency with a special ink. Currenry printed with the ink would change color depending on how ordinary light strikes it, whereas photocopied counterfeits of such crurensy would not. Because this technique would allow anyone to detect photocopied counterfeit currency easily, it should be adopted instead of microprinting, despite the exp€nse.

Which one of the following, if true, provides the most support for the recommendation made by the argument?

(A)

(B) (C) (D) (E)

When an anticounterfeiting technique depends on the detection of counterfeits by experts, the cost of inspection by experts adds significantly to the cost to society of that technique. For any anticounterfeiting technique to be effective, the existence of anticounterfeiting techniques should be widely broadcast, but the method by which counterfeits are detected should be kept secret. The process of microprinting paper currency involves fewer steps than does the printing of paper currency with the special ink. Before photocopying technology existed, most counterfeits of paper currency were accomplished by master engravers. Many criminals do not have access to the advanced photocopiers that are needed to produce counterfeits of microprinted paper currency that cashiers will accept as real.

broadcast journalism,

Which one of the following, if true, most seriously undermines the defense offered by the dean? (A) (B)

(c) (D) (E)

More than half of the school's students came from jobs in journalism to improve their skills. Some newspaper editors do not regard journalism schools as a necessary part ofthe training of a journalist. The number of cities with more than one major newspaper has declined sharply over

the last 25 years. The program offered by the Hyperion School of ]ournalism is similar in quality and content to those offered by its peer institutions. The proportion of applicants to the Hyperion School of Journalisrn that are admitted is lower than it rvas ten years ago.

248

Sources; PrcpTest 27, Section I, Questlon 70 PrcpTest 26, Sectlon 2, Questlon 5 PrcpTest 25, Sectlon 2, Questlon 4

PDF compression OCR web optimization using a watermarked evaluation copy of CVISION PDFCompressor  

Stre ngthe n/Wea ke n Qu esti o n s

59.

Some plants have extremely sensitive biological thermometers. For example, the leaves of

rhododendrons curl when the temPerature of the air around them is below OoC (Celsius)' Similarly' mature crocus blossoms open in temperatures above 2"C. So someone who simultaneously observed rhododendroas with uncurled leaves, crocuses with unopened blossoms, and a thermometer mature but showing loC could determine that the thermometer's reading was accurate to within plus or minus loC.

Which one of the following, if true, most seriously undermines the reasoning above?

(A)

Neither rhododendrons nor crocuses bloom for more than a few weeks each year, and the blossoms of rhododendrons growing in any

area do not appear until at least several weeks after crocuses growing in that area haYe

(B) (C) (D)

(E)

ceased

to bloom.

Many people find it unpleasant to be outdoors for long periods when the temperature is at or about 1"C. The climate and soil conditions that favor the growth of rhododendrons are also favorable to the growth of crocuses.

Airtemperature surroundingrhododendrons, which can grow 12 feet tall, is likely to differ from air temperature surrounding crocuses' which are normally only a few inches high, by more than 2oC, even if the two plants are growing side bY side. Certail types of thermometers that are commonly used to measure outdoor

temPeratures can be extremely accurate in moderate temperature ranges but much less accurate in warmer or colder temperature ranges.

50.

The authors of a recent article examined warnings of an impending wave of extinctions of animal species within the next 100 years. These authors saythat no evidence exists to support the idea that the rate of extinction of animal species is now accelerating. They are wrong, however. Consider only the data on fishes: 40 species and subspecies of North American fishes have the27 twentieth century, 13 between 1900 vanished and 1950,inand since 1950.

The answer to which one of the following questions would contribute most to an evaluation of the argument?

(A)

(B)

Were the fish species and subspecies that became extinct unrepresentative of animal species in general with regard to their pattern

of extinction? How numerous were the populations in 1950

ofthe species and subspecies ofNorth

American fishes that have become extinct

(C) (D) (E)

since 1950? Did any of the species or subspecies of North American fishes that became extinct in the twentieth century originate in regions outside

of North America? What proportion of North American fish species and subspecies whose populations were endangered in 1950 are now thriving? Were any of the species or subspecies of North American fishes that became extinct in the twentieth century commercially important?

6t. Theplay Mankindmust

have been written between 1431 and l47l.It cannot have been written before 143 1, for in that year the rose noble, a coin mentioned in the play, was first circulated. The play cannot have been written after 147I, since in that year King Henry VI died, and he is mentioned as a

living monarch in the play's dedication.

The argument would be most seriously weakened which one of the following were discovered?

(A)

(B) {C) (D) (E)

if

The Royal Theatre Company includes the play on a list of those performed in 1480. Another coin mentioned in the play was first

minted inl422.

The rose noble was neither minted nor circulated after 1468. Although HenryVI was deposed in 1461, he was briefly restored to the throne in 1470. In a letter written in early 1428, a merchant told of having seen the design for a muchdiscussed new coin called the "rose noble."

Sourcesi Preplest 25, Eection Preplest 24, Sectlon PrepTest 24, Seetion

2, Questlon t5 2, Questlon 76 3, Questlon 9 I(APLAN

249

)^(

*

F

o ct' n* flgl

ET

vo  lule -Lt 5

OQ

PDF compression OCR web optimization using a watermarked evaluation copy of CVISION PDFCompressor  

ISAT MasterY Proctice The local agricultural official gave the fruit growers of the District 10 Farmers' Cooperative a new pesticide that they applied for a period of three.years io their pear orchards in place of the pesticide they had formerly applied' During those three years, the proportion of piars lost to insects was significantly iess-than it had been during the previous three-year

62.

results, the official period. On the basis of these concluded that the new pesticide was more effective than the old pesticide, at least in the short term, in limiting the loss of certain fruit to insects' The official's conclusion is most strongly supported if which one of the following groups of trees did not show a reduction in losses offruit to insects?

(A)

)x

(B)

*

(C) (D)

bto .=

tr

0 IA

(E)

G

uo

rI

G

.g

bo

o

J

63.

peach trees grown in the district that were

treated with the new pesticide instead of the old Pesticide peach trees grown in the district that were treated with the new pesticide in addition to the old Pesticide pear trees grown in the district that were treated with the old pesticide instead of the new pesticide pear tiees grown in a neighboring district that were treated with neither the old nor the new pesticide grown in a neighboring district that pear -weretrees treatid with the new pesticide instead of the old Pesticide

64.

Brorvnlea's post office must be replaced lvith a larger one. The present one cannot be expanded. Land near the present location in the center of town is more expensive than land on the outskirts of town' Since the cost of acquiring a site is a significant part of the total construction cost, the post of{ice clearly could be built more cheaply on the outskirts of town'

Which one of the follot'ing, if true, most seriously undermines the argument's stated conclusion?

(A)

(B)

The nerv post office will have to be built in accordance with a demanding new citl'wide building code. If the new post office is built on the outskirts

of town, it will require a parking lot, but

(D)

(E)

If the new post office is built near the center of town, disruptions to city traffic rvould have to

(C)

be minirnized by taking such steps as doing some construction work in stages at night and on weekends.

A member of the British Parliament is reputed to have said, "The first purpose of good social reform is to increase the sum total of human happiness' So, any reform which makes somebody happy is achieving its purpose. Since the reform I propose would mike my constituents happy' it is a good social reform." Which one of the following, if true, most seriously weakens the argument attributed to the member of Parliament?

(A) (B)

(C) (D) (E)

Different things make different people hapPy. The proposed reform would make a few people huppy, but would not increase the happiness of most other peoPle. The proposed reform would affect only the member of Parliament's constituents and would make them haPPY. Increasing some people's happiness might not increase the sum total of human happiness if others are made unhaPPY. Good social reforms usually have widespread suPPort'

Sources; PrcpTest 22t Sectlon 2, Question PrcpTest 22, Sectlon PrepTest 21, Section

254

if

sited near the present post office it will nol If the new post office is built on the outskirts of town, current city bus routes will have to be expanded to provide access' If the new post office is built on the outskirts of torvn, residents will make decreased use of post of;fice boxes, with the result that mail carriers will have to deliver more mail to homes.

I

4, Question 26 2, Question 7

PDF compression OCR web optimization using a watermarked evaluation copy of CVISION PDFCompressor  

St r e n g t h e n/We

65.

Eva:

A "smart highway" system should be installed, one that would monitor areawide trafBc patterns and communicate with computers in vehicles or with programmable highlvay signs to give drivers information about traffic congestion and alternate routes' Such a system, we can infer, wouki result in around cities that improved flow and traffic in would do more than improve drivers'tempers; it would decrease the considerable loss of money and productivity that now results from traffic congestion'

56.

An ingredient in marijuana known as THC has been found to inactivate herpes viruses in experiments' ln previous experiments researchers found that inactivated herpes viruses can convert healthy cells into cancer cells. It can be concluded that the use of marijuana can cause cancer' weakens the argument? Several teams of scientists performed the various experiments and all of the teams had

(A)

similar results.

(B)

There are already traffic reports on the radio. Why would a "smart highway" system be anY better?

'Ihe carcinogenic effect of THC could be

neutralized by the other ingredients found in marijuana. When THC kills herpes viruses it weakens the immune system, and it might thus diminish the body's ability to fight other viruses, including viruses linked to cancers' If chemists modifu the structure of THC' THC can be safely incorporated into medications to prevent herPes. To lessen the undesirable side effects of chemotherapy, the use of mariiuana has been

(C)

If Eva responded to l.uis by saying that the current

one-minute radio reports ate too short to give a sufficient description of overall patterns of traffic congestion, whiih one of the following, if true, rvould

(D)

(A)

iE)

mos't strengthen Luis's challenge?

(B) (C) (D) (E)

Bad weather, which radio stations report' would cause traffic to slow- down whether or was in

than to install a "smart highway" systemRadio reports can take note of congestion once it occurs, but a "smart highway" systern could anticipate and forestall it in many instances' The proposed traffic monitoring would not reduce the Privary of drivers. Tbll collection booths, which constitute traffic bottlenecks, would largely be replaced in the "smart highway" system by electronic debiting of commuters' accounts while traffic proceeded at full sPeed.

4 ayqtt ig :

Which one of the following, if true, most seriously

Luis:

a "smart highway" system not operation. It would be less costly to have radio stations that give continual, lengthier traffic reports

ake

* *

0

recommended for cancer patients who are free of the herpes virus.

67.

OQ

tt is probably not true that colic in infants is caused by the inability of those infants to tolerate certain antibodies found in cow's milk, since it is often the case that symPtoms of colic are shorvn by infants that are f'ed breast milk exclusivelY' Which one of the following, if true, most seriously weakens the argument?

(A)

(B) (C) (D)

(E)

A study involving 500 sets of twins has found that if one infant has colic, its tlvin will probably also have colic. Symptoms of colic generally disappear as infants grow older, whether the infants have been fed breast milk exclusively or have been fed infant formula containing cow's milk' In a study of 5,000 infants who were fed only infant formula containing cow's milk, over 4,000 of the infants never displayed any symPtoms of colic. When mothers of infants that ate fed only breast milk eliminate cov/s milk and all products made from cow's milk from their own diets, any colic symptoms that their infants have manifested quickly disappear. Infants that are fed breast milk develop mature digestive systems at an earlier age than do those that are fed infant formulas, and infants with mature digestive systems are better able to tolerate certain proteins and antibodies found in cow's milk.

Sources: PrepTest 21, Sectlon 3, Questlon

F

I

PrcpTest 20, Sectlon 7, Questlons 4 and 72

251

-I r'r

ol

-fr

o sl

f,l

o :l II

=e

PDF compression OCR web optimization using a watermarked evaluation copy of CVISION PDFCompressor  

68.

Garbage dumps do not harm wildlife. Evidence is furnished by the Masai-Mara reserve in Kenya, where baboons that use the garbage dumps on the reserve as a food source mature faster and have inore offspring than do baboons on the reserve that do not scavenge on garbage. Each of the following statements,

if true, casts doubt

on the argument EXCEPT:

(A)

(B) (C)

* *

(D) (E)

blo

.=

tr

o

t'

rE

o

& r o I

II

a0

o J

69.

The baboons that feed on the garbage dump are of a different species from those that do

not.

The life expectancy ofbaboons that eat garbage is significantly lower than that of baboons that do not eat garbage. The cholesterol level of garbage-eating baboons is dangerously higher than that of baboons that do not eat garbage. The population of hyenas that live near unregulated garbage landfills north of the reserve has doubled in the last two years. The rate ofbirth defects for the baboon population on the reserve has doubled since the first landfills were opened.

Babies who can hear and have hearing parents who expose them to speech begin to babble at a certain age as a precursor to speaking. In the same way, deaf babies with deaf parents who communicate with them and with each other by signing begin to babble in signs at the same age. That is, they make repetitive hand gestures that constitute, within the language system of signs, the analogue of repeated syllables in speech.

70.

A favored theory to explain the efiinction of dinosaurs, together with many other species, has been the globally catastrophic collision of a large asteroid with the Earth. Supporting evidence is an extraterrestrial chemical element in a layer of dust found worldwide at a geological level laid down contemporaneously with the supposed €yent. A new contends any asteroid competing theory irrelevant, was it wasthat because massive volcanicimpact activity that caused the extinctions by putting enough dust into the atmosphere to cool the planet. The Deccan region of India contains extensive volcanic flows that occurred within the same time period as the supposed asteroid impact and the

extinctions.

Which one of the following, if true, most strongly indicates that the asteroid-impact theory is at least incomplete, if not false?

(A) (B)

(C)

(D)

The information above, if accurate, can best be used as evidence against which one of the following hypotheses?

(A) (B)

(C) (D) (E)

Names of persons or things are the simplest words in a language, since babies use them before using the names of actions or Processes, The development of language competency in babies depends prirnarily on the physical maturation of the vocal tract, a process that requires speech-oriented vocal activity. In the absence of adults who communicate with each other in their presence, babies develop idiosyncratic languages. In babbling, babies are unalvare that the sound or gesture combinations they use can be employed in a purposive way. The making of hand gestures by hearing babies who have hearing parents should be

(E)

Large concentrations of dinosaur nests with

fossil eggs found in Alberta indicate that at least some species of dinosaurs congregated in large groups during some part of their lives. Dinosaur remains indicate that some species of dinosaur could have migrated in herds over wide ranges, so that they could have traveled to escape the local effects of certain catastrophes. Legends from many cultures, such as the Greek legend that Cadmus raised an army by sowing dragons'teeth in the ground, show that various ancient peoples worldwide were familiar with the fossils of dinosaurs. In the Gobi desert in China, where now only small animals can eke out an existence, fossil dinosaur skeletons 27 feet long were found in circumstances indicating that the climate there was as dry when the dinosaurs lived as it is now. The fossil record in Montana from below the layer of extraterrestrial dust shows a over time in dinosaur species diminution from 35 to 13, and dinosaur teeth found above rhe dust layer show a diminution in species from l3 to 5.

interpreted as a part of their developing language.

Sources; PrcpTest 2A, Sectlon 4, Questlon 23 PrcpTest PrepTest

252

17, Sectlon 3, Questlon t 5 tO, Sectlon 1, Questlon 6

PDF compression OCR web optimization using a watermarked evaluation copy of CVISION PDFCompressor  

Str e n gth e n /Wea ke n Qu e sti o n s

7t,

Data from satellite photographs of the tropical rarn forest in Melonia show that last year the deforestation rate of this environmentally sensitive zone was significantly lower than in previous years. The Melonian goYernment, which spent millions of dollars last year to enforce laws against burning and cutting of the forest, is claiming that the satellite data its increasedeffective' indicate thatare proving efforts to halt the destruction

Which one of the following, if true, most seriously undermines the government's claim?

(A)

(B) (C) (D) (E) 72.

Landowner opposition to the government's antideforestation efforts grew more violent last year in response to the increased enforcement.

Rainfall during the usually dry 6-month annual burning season was abnormally heavy Government agents had to issue fines totaling over $9 nillion to 3,500 violators of burning-and-cutting regulations. The inaccessibility of much of the rain forest has made it impossible to confirm the satellite data by direct observation from the field. Much of the money that was designated last year for forest preservation has been spent on research and not on enforcement.

Which one of the following if true, most seriously weakens the argument?

(B) (C) (D) (E)

H:

The group of works exhibited in this year's Metropolitan Art Show reveals a bias in favor of photographers. Equal numbers of photographers, sculptors, and painters submitted works that met the traditional criteria for the show, yet more photographs were exhibited than either sculptures or paintings. As you know, each artist was allowed to submit work in one medium on1y.

How could there have been bias? All submitted works that met the traditional criteria-and only those vv61l6s-1vsrs exhibited in the show

Which one of the following, if true, most strongly supports G's allegation of bias?

(A)

last year.

Purebred dogs are prone to genetically determined abnormalities. Although such abnormalities oft en can be corrected by surgery, the cost can reach several thousand dollars. Since nonpurebred dogs rarely suffer from genetically determined abnormalities, potential dog owners who want to reduce the risk of incurring costly medical bills for their pets would be well advised to choose nonprrEbred dogs'

(A)

73. G:

(B) (C)

(D)

(E)

If an artist

has had one of his or her works exhibited in the Metropolitan Art Shoq that artist has an advantage in getting commissions and selling works over artists who have never had a work exhibited in the show. The fee for entering photographs in the Metropolitan Art Show was $25 per work submitted, while the fee for each painting or sculpture submitted was $75. The committee that selected from the submitted works the ones to be exhibited in this year's Metropolitan Art Show had four members: one photographer, one sculptor, one painter, and one who worls in all three media but is the least known of the four members. Reviews of this year's Metropolitan Art Show that appeared in major newspapers and magazines tended to give more coverage to the photographs in the show than to the sculptures and paintings that were exhibited. In previous years, it has often happened that more paiatings or more sculptures were exhibited in the Metropolitan Art Show than photographs, even though the total number of works exhibited each year does not vary widely.

JL

)^(

Most genetically determined abnormalities in dogs do not seriously affect a dog's general well-being. All dogs, whether purebred or nonpurebred, are subject to the same common nongenetically determined diseases. Purebred dogs tend to have shorter natural life spans than do nonpurebred dogs. The purchase price of nonpurebred dogs tends to be lower than the purchase price of purebred dogs' A dog that does not have genetically determined abnormalities may nevertheless have offspring with such abnormalities.

Sources: PrepTest 9, Section PrepTest 9, Section

2, Questlons 4 and 72 4, Question 77 I(APLAN

2s3

F

o qg. t' ol, It

n

o $ tt c5 IT =e

PDF compression OCR web optimization using a watermarked evaluation copy of CVISION PDFCompressor  

LSAT

74.

Mastery Proctice

High-technology medicine is driving up the nation,s health care costs. Recent advances in cataract surgery illustrate why this is occurring. Cataracts are a mijor cause of blindness, especially in elderly people. Ten years ago, cataract surgery \^/as painfrrl and not always effective. Thanl$ to the new technology used in cataract surgery, the operation now restores vision dramatically and is less expensive. These two have caused the numtrer of cataract operationsfactors performed to increase greatly, which has, in turn, driven up the total amount spent on cataract surgery. Each of the following,

if true, would support

76.

the yeri, arr apelike creature supposedly eiistirig in the Himalayas, does not exist 6C.uur. ihere haie been no scientifically confirmed sightings. A verifiable sighting of the yeti would proie that the does exist, but the absence ofsightings creature cannot prove that it does not.

Which one of the following considerations, if true, best counters the argumentl

(A)

a

challenge to the author's explanation ofthe increase in the number of cataract operations EXCEpT:

(A)

(B)

)^(

*

(C)

00

(n) . (E)

'=s

o- 75 tt tg

go

r

The overall population of the nation has increased from what it was ten years ago. Any one individual's chance of developing cataracts is greater than it was ten years ago. The number of older people has increased during the last ten y€ars. Today, health insurance covers cataract surgery for more people than it did ten years ago. People who have had unsuccessfi.rl cataract

(B) (C)

surgery are left with more seriously impaired vision than they had before the surger),.

-l

IE

lr,

'fo o J

Valitania's long-standing practice of paying high salaries to its elected politicians has had a disastrous effect on the level of integrity among politicians in that co-untry. This is because the prospect of earning a high salary always attractive to anyone whose primary is airn in life is to make money, so that ineyitably the wrong people must have been attracted into Valitanian politics: people who are more interested in making money than in serving the needs of the nation.

Which one of the following, if true, would weaken the argument?

(A)

(B) (C) (D) (E)

Many Valitanian candidates for elected ofiice spend some of their own money to ffnance their campaigns. Most Valitanian elective offices have four-year terms. No more people compete for elected office when ofliceholders are paid well than when they are paid poorly. Only politicians who rely on their offices for income tend to support policies that advance their own selfish interests. Most of those who are currentlyValitanian politicians could have obtained better-paid work outside politics.

Arguing that there was no trade between Europe and East Asia in the early Middle Ages because theie are no written records of such trade is like arluing that

(D) (E)

77.

Auto industryexecutive: Statistics showthat cars that were built smaller after 1977 to make them more fuel-efficient had a higher incidence of accident-related fatalities thin did their earlier, larger counterparts. For this reason we oppose recent guidelines that would require us to produce cars with higher fuel efficiency,

Which one of the following, if true, would constitute the strongest objection to ihe executive,s argument?

(A)

(B) (C) (D)

6, Sectlon 2, euesilons T and Preplesl 8, Sectlon 3, Question 13

Sources: PrepTest

PrepTest 5, Sectlon

254

,

euestlon

0

tg

Most of the evidence for the existence of trade between Europe and East Asia in the early Middle Ages is archaeological and therefore does not rely on written records, Although written records of trade in East Asia in the early Middle Ages survived, there are almost no European documents from that period that mention trade at all. Any trade between Europe and East Asia in the early Middle Ages would necessarily have been of very low volume and wodd have involved high-priced items, such as precious metals and silk. There have been no confirmed sightings of the yeti, but there is indirect evidenie, suih a, footprints, which if it is accepted as authentic would establish the yeti's existence. There are surviving E.r.op"un and East Asian written records from the early Middle Ages that do not mention trade between the tio regions but would have been very likely to do so if this trade had existed.

(E)

Even after 1977,large automobiles were frequently involved in accidents that caused

death or serious iniury. Although fatalities in accidents involving small cars have increased since 1977, the number of

accidents has decreased. New computerized fuel systems can enable large cars to meet fuel efficiency standards established by the recent guideiines. Modern technology can muke small cars more fuel-efficient today than at any other time in

their production history.of larse Fuel efficienry in modeli cars rose immediately after 1977 but hai been declining eyer since.

PDF compression OCR web optimization using a watermarked evaluation copy of CVISION PDFCompressor  

Stre n gth e n/We 78.

As air-breathing manimals, whales must once have lived on land and needed hind limbs capable of supporting the mammals'rveight. Whales have the bare remnants of a pelvis. If anirnals have a pelvis, we expect them to have hind limbs. A nervly discovered fossilized whale skeleton has very fragile hind limbs that could not have supported the animal's rveight on

skeleton had a partial pelvis. land. This If the statements above are true, ivhich one of the following, if also true, rvould most strongly support the conclusion that the fragile hind limbs are remnants of limbs that land-drvelling rvhaies once had?

{A) (B) (C) (D)

Whale bones older than the fossilized hind limbs confirm that ancient whales had full pelvises. No skeletons of ancient whales with intact hind limbs capable of supporting the mammals'weight have ever been found. Scientists are uncertain whether the apparently nonfunctioning lirnbs of other early mammals derived from once-functionirlg limbs of their ancestors. Other large-bodied mammals like seals and sea

lions maneuver on beaches and rocky coasts

(E)

without fully functioning hind limbs.

Some smaller sea-dwelling mammals, such as modern dolphins, have no visible indications

of hind limbs.

79.

a

ke

n

Qu e sti o n s

Older United States automobiles have been identified contributing disproportionately to global air pollution. The requirement in many iurisdictions that automobiles pass emission-control inspections has had the effect of taking many such automobiles out of service in the United States, as thev fail inspection and their owners opt to buy newer automobiles. Thus the burden of pollution such older United States automobiles contribute to the global atmosphere rvill be gradually reduced over the as

next decade.

Which one of the following, if true, most seriously rveakens the argument?

(A)

(B) (C) (D) (E)

It is impossible to separate the air of one country or jurisdiction from that of others,

since air currents circle the globe. When automobiles that are now new become older, they will, because of a design change, cause less air polh"rtion than older automobiles do now. There is a thriving market for used older United States automobiles that are exported to regions that have no emission-control

* *

F

o q3.

regulations.

fhe number of jurisdictions in the United

States requiring automobiles to pass emission-control inspections is no longer

increasing. Even if all the older automobiles in the United States were retired from service, air pollution from United States automobiles could still increase if the total number of automobiles in use should increase significantly.

f.t g

7

o  t tr 0

=.

oe

Soutce: nepTest 5, Section

3, Questions 6 and 8

255

PDF compression OCR web optimization using a watermarked evaluation copy of CVISION PDFCompressor  

|SAT Mastery Pradice

80.

Antarctic seals dive to great depths and stay submerged for hours. They do not rely solely on oxygen held in their lungs, but also store extra

oxygen in their blood. Indeed, some researchers hypothesize that for long dives these seals also store oxygenated blood in their spleens. Each of the following, if true, provides some support for the researchers' hypothesis EXCEPT:

(A) (B)

(C)

* *

(D) (E)

)^( ulD

tr g

TI

o 1n a {u

E

IT

G

.g

u0

o

-l

81.

Horses are known to store orygenated blood in their spleens for use during exertion. Many species of seal can store oxyg€n directly in their muscle tissue. The orygen contained in the seals'lungs and bloodstream alone would be inadequate to support the seals during their dives. The spleen is much larger in the Antarctic seal than in aquatic mammals that do not make long dives. The spleens ofAntarctic seals contain greater concentrations of blood vessels than are contained in most of their other organs.

A l99l calculation was made to determine what, if any, additional health-care costs beyond the ordinary are borne by society at large for people who live a sedentary life. The figure reached was a lifetime average of $1,650. Thus people's voluntary choice not

82. Before

1986 physicists believed they could describe the universe in terms of four universal forces. Experiments then suggested, however, a flfth universal force of mutual repulsion between particles of matter. This fifth force would explain the occurrence in the experiments of a smaller measurement of the gravitational attraction between

bodies than the established theory predicted. Which one of the following, if true, most strengthens the argument that there is a fifth universal force?

(A)

(B) (C) (D) (E)

The extremely sophisticated equipment used for the experiments was not available to physicists before the 1970s. No previously established scientific results are incompatible with the notion of a fifth universal force. Some scientists have suggested that the alleged fifth universal force is an aspect of gravity rather than being fundamental in itself. The experiments were conducted by physicists in remote geological settings in which factors affecting the force of gravity could not be measured with any degree of precision. The fifth universal force was postulated at a time in which many other exciting and productive ideas in theoretical physics were developed,

to exercise places a significant burden on society.

Which one of the following, if true and not taken into account by the calculation, most seriously weakens the argument?

(A)

(B) (C) (D) (E)

Many people whose employment requires physical exertion do not choose to engage in regular physical exercise when they are not at work. Exercise is a topic that is often omitted from discussion between doctor and patient during a patient's visit. Physical conditions that eventually require medical or nursing-home care often first predispose a person to adopt a sedentary

lifestyle. Individuals vary widely in the amount and kind of exercise they choose, when they do

exercise regularly. A regular program ofmoderate exercise tends to increase circulation, induce a feeling of well-being and energy, and decrease excess

weight.

$ourcos.' Prcplest 37, Sectlon FrcpTast 38, Sectlon Prcp|est 36, Sectlan

2, Questlon 20

t, Questlon 25

3, Questlon 26

PDF compression OCR web optimization using a watermarked evaluation copy of CVISION PDFCompressor  

Stren gth en/Wea ke

83.

Parent

P:

84.

Children rvill need computer skills to deal rvith tomorrow's lvorld' Computers should be introduced in kindergarten, and computer languages should tre required in high school'

Parent

That would be pointless' Technology advances so rapidly that the computers used by today's kindergartners and the comPuter

@

Politician: All nations that place

-

-

in today's high schools would

their voice in world affairs.

Which one of the following, if true, is the strongest logical counter parent P can make to parent Q's objection? (A) When technology is advancing rapidly. regular training is necessary to keep one's skills at a level proficient enough to deal with the societY in which one lives' (B) Throughout history people have adapted to change, and there is no reason to believe that today's children are not equally capable of adapting to technology as it advances' (C) In the process of learning to work with any comPuter or computer language, children

Each of the following,

argument

(A)

if

a

nation

(B)

(c)

incriase technologY. computertheir Automotive technology is continualla advancing too, but that does not result in one's having to relearn to drive cars as the new advances are incorporated into new automobiles' Once people have graduated from high school' they have less time to learn about computers and technology than they had during their

EXCEPT:

advantageous position. Those naiions that lose influence in the world community do not necessarily suffer from a threat to their value system or way of life. Allowing one's country to lose its technological edge, especially as concerns rveaponry, would be foolish rather than merelv a historical accident.

(D) (E)

85.

if true, weakens the politician's

The top level of taxation must reach 45 percent befori taxation begins to deter inventois and industrialist from introducing new technologies and industries. Making a great deal of money is an insignificant factor in driving technological innovation. Falling behind in the international arms race does not necessarily lead to a strategically less

ability to interact with

schooling Years.

So

wants to maintain its value system and way of life, it must not allow its highest tax bracket to exceed 30 Percent of income'

adults.

(E)

high tax on

income produce for technological innovation, and all nations in which technological innovation is hampered inevitably fall behind in the international arms race, Those nations that, through historical accident or the foolishness of their political wind up in a strategically leadership, disadvantageous position are destined to lose

becimi obsolete by the time these children are

(D)

Questi o n s

thereby a negative incentive

Q:

languages taught

a

n

__A._

)^( ft

oa o

We can learn about the living conditions of a vanished culture by examining its language. Thus, it is likely that the people who spoke Proto-IndoEuropean, the language from which all IndoEuropean languages descended, lived in a cold climate. isolated from ocean or sea, because ProtoIndo-European lacks a word for "seal'yet contains ttwolf.tt ttwinter," t'snowr" and words for Which one of the follorving, if true, most seriously weakens the argument?

(A)

(B) (C) (D) (E)

A word meaning "fish"'was used by the people who spoke Proto-Indo-EuroPean. Some languages lack words for prominent elements of the environments of their speakers. There are no known languages today that lack a word for "sea." Proto-Indo-European possesses words for

"hgat."

The people who spoke Proto-Indo-European were nomadic.

gources: PrepTest 35, Sectlon 7, Questlon 4 Preplest 35, Section 4, Question t7 PrepTest

F

29, Section 7, Quesilon 76

257

.l

-r EI |

fr o $

o o :t II

=q

PDF compression OCR web optimization using a watermarked evaluation copy of CVISION PDFCompressor  

ISAT Mastery

Prodice

86. Marianne

is a professional chess player who hums audibly while playing her matches, thereby

Questions 87-88

distracting her opponents. When ordered by chess officials to cease humming or else be disqualified from professional chess, Marianne protested the order. She argued that since she was unaware ofher humming, her humming was involuntary and that

The fishing industry cannot currently be relied upon to help the government count the seabirds killed by net fishing, since an accurate count might result in restriction of net fishing. The government should therefore institute a program under which tissue samples from the dead birds

therefore she should not be heid responsible for it. Which one of the following, if true, most undermines

are to determile amount of toxins in thea industry would then have birds. The the eaten by the fish examined

(A)

with toxins.

Marianne's argument against the order?

(B)

* * *

(c) (D) (E)

bo .E

The officials of chess have little or no authority to control the behavior of its professional players outside of matches' Many of the customs of amateur chess matches are not observed by professional chess players. Not all of a person's involuntary actions are actions of which that person is unaware' A person who hums involuntarily can easily learn to notice it and can thereby come to

reason to turn in the bird carcasses, since the industry needs to know whether the fish it catches are contaminated

87.

control it. Not all of Marianne's opponents are distracted by her humming during chess matches'

the argument?

(A)

The seabirds that are killed by net fishing do not eat all of the species of fish caught by the

(B)

The government has not in the past sought to determine whether fish were contaminated with toxins by examining tissue samples of seabirds. The government cannot gain an accurate

tr

(D)

tu

(E)

{r g,

tg

t 'f,o o J

fishing industry.

(c)

o ut

I

Which one of the follolving, if true, most strengthens

88.

the number by net count of seabirds the fishing industrykilled cooperates. fishingofunless If the government knew that fish caught by the fishing industry were contaminated by toxins, the government would restrict net fishing. If net fishing were restricted by the government, then the fishing industry would become more inclined to reveal the number of seabirds killed by net fishing.

Which one of the following, if true, most strongly indicates that the government program would not by itself provide an accurate count of the seabirds killed by net fishing?

(A)

(B) (C) (D) (E)

The seabirds killed by net fishing might be contaminated with several different toxins evenfishing eat could only one kind of fish.the if the birds The learn whether industry fish it catches are contaminated with toxins if only a few of the seabirds killed by the nets were examined. The government could gain valuable information about the source of toxins bv examining tissue samples of the seabirds caught in the nets. The fish caught in a particular net might be contaminated with the same toxins as those in the seabirds caught in that net. The government would be willing to certify that the fish caught by the industry are not contaminated with toxins if tests done on the seabirds showed no contamination.

Sources; PrepTest 20, Sectlon 4, Questlon 25 PrepTest

?

qR

I(AP tAN,)

77, Sectlon 2, Questlons

7 and

I

PDF compression OCR web optimization using a watermarked evaluation copy of CVISION PDFCompressor  

Stre n gth e n/Weo ke 89.

Waste management companies, which collect waste for

disposal in landfills and incineration plants, report thal disposable plastics make up an ever-increasing percentage of the waste they handle. It is clear that attempts to decrease tle amount of plastic that people throw away in the garbage are failing.

Which one of the following, if true, most seriously weakens the argument? Because plastics create harmfirl pollutants when burned, an increasing perceatage of the plastics handled by waste management companies are being disposed of in landfills. Although many plastics are recyclable, most of the plastics disposed of by waste management companies are not. (c) People are more likely to save and reuse plastic

(A)

(B)

(D)

(E)

containers than containers made of heavier materials like glass or metal. An increasing proportion of the paper, glass, and metal cans that waste management companies used to handle is now being

90.

Qu esti o n s

When butterfat was considered nutritious and healthful, a law was enacted requiring that manufacturers use the term "imitation butter" to indicate butter whose butterfat content had been diminished through the addition of water. Today, it is known that the high cholesterol content of butterfat makes it harmful to human health. Since the public should be encouraged to eat foods with lower rather than higher butterfat content and since the term "imitation" with its connotations of falsity deters many people from purchasing products so designated, manufacturers who wish to give reduced-butterfat butter the more appealing name of "lite butter" should be allowed to do so.

Which one of the following, undermines the

(A) (B)

recycled.

if

argument?

true, most seriously

fi'

The manufacturers who prefer to use the word Sb "lite" instead of "imitatiorl'are motivated ? principally by the financial interest of their tk stockholders. The manufacturers rvho wish to call their product "lite butter" plan to change the composition of the product so that it contains

F

While the percentage of products using plastic packaging is increasing, the total amount of plastic being manufactured has remained unchanged.

n

(c)

(D)

more water than it now does. Some individuals who need to reduce their

intake of cholesterol are not deterred from using the reduced-butterfat product by the negative connotations of the term "irnitation." Cholesterol is only one of many factors that contribute to the types of health problems

with which the consumption of

(E)

excessive

amounts of cholesterol is often associated. Most people deterred from eating "imitation butter" because of its name choose alternatives with a lower butterfat content than this product has.

Soutces.' PrepTest 9,5eetion Prcplest 9, Section

2, Question 7 4, Questlon 22

259

o E. fl o ;

o

sl |a

o rl -

5'

ocl

PDF compression OCR web optimization using a watermarked evaluation copy of CVISION PDFCompressor  

ISAT Mastery Pradice

91.

* * * a0 .E

tr

o I, o o

&

I.

IE

Samples from the floor of a rock shelter

in

Pennsylvania were dated by analyzing the carbon they contained. The dates assigned to samples associated with human activities formed a consistent series, beginning with the present and going back in time, a series that was correlated with the depth from which the samples came. The oldest and deepest

92.

Two paleontologists, Dr. Tyson and Dr. Rees, disagree over the interpretation of certain footprints that were left among other footprints in hardened volcanic ash at site G. Dr. Tyson claims they are clearly early hominid footprints since they show human characteristics: a squarish heel and a big toe immediately adjacent to the next toe. However, since

sample was dated at 19,650 years before the present, plus or minus 2,400 years. Skeptics, viewing that date as too early and inconsistent with tJre accepted date of human migration into North America, suggested that the samples could have been contaminated by dissolved "old carbon" carried by percolating groundwater from nearby coal deposits.

the footprints indicate that if hominids made those prints they would have had to walk in an unexpected cross-stepping manner, by placing the left foot to the right of the right foot, Dr. Rees rejects Dr. Tysorls

Which of the following considerations, if true, argues most strongly against the suggestion of the skeptics?

(A)

(A)

(B) (C) (D) (E)

I E bo o J

260

No likely mechanism of contamination involving percolating groundwater would have affected the deeper samples from the site without affecting the uppermost sample. Not every application of the carbon-dating procedure has led to results that have been generally acceptable to scientists. There is no evidence that people were using coal for fuel at any time when the deepest layer might have been laid down. No sample in the series, when retested by the carbon-dating procedure, was assigned an earlier date than that assigned to a sample

from a layer above it. No North American site besides the one in

Pennsylvania has ever yielded a sample to which the carbon-dating procedure assigned date that was comparably ancient.

,gp

conclusion.

Which one of the following, if true, most seriously undermines Dr. Tyson's conclusion?

(B) (C) (D) (E)

The footprints showing human characteristics were clearly those of at least two distinct

individuals. Certain species of bears had feet very like human feet, except that the outside toe on each foot was the biggest toe and the innermost toe was the smallest toe. Footprints shaped like a human's that do not show a cross-stepping pattern exist at site M, which is a mile away from site G, and the two sets of footprints are contemporaneous. When the moist volcanic ash became sealed under additional layers of ash before hardening, some details of some of the

footprints were erased.

Most of the other footprints at site G were of animals with hooves.

a

Sources: PrepTest 6, Section 3, Questlon t7 Preplest 5, Sectlon t, Questlon 8

PDF compression OCR web optimization using a watermarked evaluation copy of CVISION PDFCompressor  

Str e n gth e n/Tlleo ke n Qu esti o ns

93.

vflp

There are about 75 brands of microwave poPcotn on the market; altogether, they account for a little over haH of the money from sales of microwave food products. It takes three minutes to pop corn in the microwave, compared to seven minutes to poP corn conventionally. Yet by weight, microwave popcorn typically costs over five times as much as

rettiion"l popcotn. |udging by the popularity of "on microwave popcorn' many people are willing to pay high price for just a little additional convenience.

a

Which one of the following statements, if true, would call into question the conclusion in the passage?

(A)

More than 50 percent of popcorn purchasers buy conventional popcorn rather than

(B)

Most people who prefer microwave Popcorn do so because it is less fattening than popcorn that is popped conventionally in oil. The price of microwave popcorn reflects its packaging more than it reflects the quality of the *popcorn contained in the package. The ratio of unpopped kernels to popped kernels is generally the same whether popcorn is PoPPed in a microwave or

(C) (D) (E)

microwave PoPcorn.

* * *

F

gq o

conventionallY in oil' Because microwave popcorn contains additives

ttrr 'fi, Ot

not contained in conventional popcorn' microwave Popcorn weighs more than an equal volume of conventional popcorn'

'H al ta

o 3 fr l'

6t

Source: PrcpTest

6, Sectlon 3, Quastlon 24

261

PDF compression OCR web optimization using a watermarked evaluation copy of CVISION PDFCompressor  

ISAT Mastery Proctice George Orwell's book 1984 has exercised much influence on a great number of this newspaper's readers. One thousand readers were surv€yed and asked to name the one book that had the most influence on their lives. The book chosen most often was the Bible; f984 was second.

94, Columnist:

ata -** -

96,

(B) (C)

* *

(D)

)^(

(E)

*

95, b'0 rrl tr ? I

o y} IE

a

*

I

TU

II

TJ

a0

o J

Age, the settlers of the Ice fust Eurasian in North probably America came from a more distant part of Eurasia.

Which one of the following, if true, most seriously

How manybooks had each person surveyed

weakens the archaeologist's argument?

read?

(A)

How many people chose boola other than 1984? How many people read the columnist's newspaper? How many books by George Orwell other than 1984 were chosen? How many of those surveyed had actually read the books they chose?

(B) (C)

Lobsters and other crustaceans eaten by humans are

(D)

more likely to conftact gill diseases wh€n se$rage contaminates their water. Under a recent proposal'

millions of gallons of local

sewage each day would be rerouted many kilometers offshore. Although this would substantially reduce the amount of sewage in the harbor where lobsters are caught, the proposal is pointless, because hardly any lobsters live long enough to be harmed by those diseases.

Which one of the following, if true, most seriously weakens the argument?

(A)

(B) (C) (D) (E)

Contaminants in the harbor other than sewage are equally harmful to lobsters. Lobsters, like other crustaceans, live longer in the open ocean than in industrial harbors. Lobsters breed as readily in sewagecontaminated water as in unpolluted water. Gill diseases cannot be detected by examining the surface of the lobster. Humans often become ill as a result of eating lobsters with gill diseases.

a North American mastodon that became extinct at the peak of

the Ice Age was recently discovered. It contains a human-made projectile dissimilar to any found in that part of Eurasia closest to North America. Thus, since Eurasians did not settle in North America until shortly before the peak

The answer to which one of the following questions would most help in evaluating the columnist's argument?

(A)

Archaeologist: A skeleton of

(E)

97.

if true, weakens the argument against the predominant theory about northern cave paintings EXCEPTT

Each of the following,

(D) Questlon 24

t,

Questlon 20

1, Questlon 8 Prepleet 35, Sectlon 4, Questlon 20 PrcpTest 33, Seetlon

252

'99

before the Ice Age were more conducive to human habitation than were those in the part of Eurasia closest to North America at that

are and there no paintings that unambiguously depict such creatures.

(C)

t,

Climatic conditions in North America just

Recently discovered prehistoric rock paintings on small islands off the northern coast of Norway have archaeologists puzzled. The predominant theory about northern cave paintings was that they were largely a description of the current diets of the painters, This theory cannot be right, because the painters must have needed to eat the sea animals populating the waters north of Norway if they were to make the long journey to and from the islands,

(B)

PrepTest 35,Section

similar projectile.

Other North American artifacts from the peak of the Ice Age are similar to ones from the same time found in more distant parts of Eurasia.

time.

(A)

Sources: PrcpTest 36,9ectlon

The projectile found in the mastodon does not resemble any that were used in Eurasia before or during the Ice Age. The people who occupied the Eurasian area closest to North America remained nomadic throughout the Ice Age, The skeleton of a bear from the same place and time as the mastodon skeleton contains a

(E)

Once on these islands, the cave painters hunted and ate land animals. Parts of the cave paintings on the islands did not survive the centuries. The cave paintings that were discovered on the islands depicted many land animals. Those who did the cave paintings that were discovered on the islands had unusually advanced techniques of preserving meats. The cave paintings on the islands were done by the original inhabitants of the islands who ate the meat of land animals.

PDF compression OCR web optimization using a watermarked evaluation copy of CVISION PDFCompressor  

98.

@

Amphibian populations are declining in numbers worldwide. Not coincidentally, the eartht ozone layer has been continuously depleted throughout the last 50 years. Atmospheric ozone blocks tfV-B, a type of ultraviolet radiation that is continuously produced by the sun, and which can damage genes. Because amphibians lack hair, hide, or feathers to shield

99.

@

Medical researcher: As expected, records covering the last four years of ten major hospitals indicate that babies born prematurely were more likely to have low birth weights and to suffer from health problems than were babies not born prematurely. These records also indicate that mothers who had received

are particularly them, theyIn UV-B gelatinous to lack the eggs radiation, addition, theirvulnerable protection of leathery or hard shells. Thus, the

care were adequate likely to have low birth prenatal weight babies thanless were mothers who had received inadequate prenatal care, Adequate prenatal care, therefore, significantly decreases the risk of low birth weight babies.

if true, would strengthen the

Which one of the followiag, if true, most weakens the

primary

cause of the declining amphibian population is the depletion of the ozone layer.

Each of the following,

argument EXCEPT:

(A) (B)

(C) (D) (E)

Of the various types of radiation blocked by

atmospheric ozone, UV-B is the only fype that can damage genes. Amphibian populations are declining far more rapidly than are the populations of nonarnphibian species whose tissues and eggs have more natural protection from lfV-B. Atmospheric ozone has been significantly depleted above all the areas of the world in which amphibian populations are declining. The natural habitat of amphibians has not become smaller over the past century. Amphibian populations have declined continuously for the last 50 years.

medical researcher's argument?

(A)

(B) (C) (D) (E)

The hospital records in&cate that many babies that are born with normal birth weights are born to mothers who had inadequate prenatal care. Mothers giving birth prematurely are routinely classified by hospitals as having received inadequate prenatal care when the record of that care is not available, The hospital records indicate that low birth weight babies were routiaely classified as having been born prematurely. Some babies not born prematurely, whose mothers received adequate prenatal care, have low birth weights. Women who receive adequate prenatal care, are less likely to give birth prematurely than ,lre women who do not receive adequate prenatal care.

* * * * €, € F,

Eq,

lr

f.| qr

 l

vG

'$}

,9.

,:ll

-r lll,.

OQ,

Sourcs,' PrcpTest 29, Sectlan

4, Quastians 20

and 24

{ll9

263

PDF compression OCR web optimization using a watermarked evaluation copy of CVISION PDFCompressor  

Mastery Prodice

ISAT

interstitial nucleus, a subregion of the brain's hypothalamus, is typically smaller for male cats than for female cats. A neurobiologist performed autopsies on male cats who died from disease X, a

100. The

disease affecting no more than '05 percent of male cats, and found that these male cats had interstitial nuclei that rvere as large as those generally found in

102. Anthropological studies indicate that distinct cultures differ in their moral codes. Thus, as long as there are distinct cultures, there are no values shared across cultures,

@ @

(A)

Which of the following statements, if true, most

(B)

(A)

* * * *

(B) (C) (D) (E)

F

0 vl I

q,

(D) (E)

cat.

The hypothalamus is known not to be causally linked to disease Y, and disease X is a subtype Y.

101. In Australia the population that is of driving age has grown larger over the last five years, but the annual nurnber of traffic fatalities has declined' This leads to

.$@ bo o

(C)

Of 1,000 autopsies on male cats rvho did not contract disease X, 5 revealed interstitial

of disease

2e J

No female cats have been known to contract disease X, which is a subtype of disease Y. Many male cats who contract disease X also contract disease Z, the cause ofwhich is unknown. The interstitial nuclei of female cats who contract disease X are larger than those of female cats rvho do not contract disease X.

nuclei larger than those of the average male

bo tr

G

argument EXCEPT:

female cats. Thus, the size of the interstitial nucleus determines whether or not male cats can contract disease X. seriously weakens the argurnent?

II

Each of the following,

the conclusion that, overall, the driving-age population of Australia consists of more skillful drivers now than five years ago. Each of the statements below,

argument EXCEPT:

if true, weakens the

(A)

Three years ago, a mandatory seat-belt law went into effect throughout Australia'

(B) (C)

Five years ago, Australia began a major road

(D) (E)

repair project. Because of increases in the price of fuel, Australians on average drive less each year than in the preceding year. The number of hospital emergency facilities in Australia has doubled in the last five years' In response to an increase in traffic fatalities, Australia instituted a program of mandatory driver education five years ago.

fuepTest

264

24, Sectlon 3, Questlon 23

Anthropologists rely on inadequate the valuestranslation of techniques to investigate cultures that use languages different from the

anthropolo gists' languages. As a result of advancing technology and global communication, we will sorneday all share the same culture and the same values. Although specific moral values differ across

cultures, more general moral principles, such as "Friendship is good," are common to all cultures. The anthropologists who have studied various cultures have been biased in favor of finding differences rather than similarities between distinct cultures. What appear to be differences in values between distinct cultures are nothing more than differences in beliefs about how to live in accordance rvith shared values.

i03. Historians of North American architecture who have studied early nineteenth-century houses with

@

rvooden floors have observed that the boards used on the floors of bigger houses were generally much narrower than those used on the floors of smaller houses. These historians have argued that, since the people for whom the bigger houses were built were generally richer than the people for whom the smaller houses were built, floors made out of narrow floortloards were probaLrly once a status symbol, designed to proclaim the owner's wealth. Which one of the following, if true, most helps to strengthen the historians' argument?

(A) (B) (C) (D) (E)

Sources; Preplest 28, Sectlon 3, Question 25 PrepTest 24, Sectlon 2, Questions 79 and 20

if true, would weaken the

More floorboards havehouses survived original big early nineteenth-century thanfrom from small early nineteenth-century houses. In the early nineieenth century, a piece of narrow floorboard was not significantly less expensive than a piece of wide floorboard of the same length. In the early nineteenth century, smaller houses generally had ferver rooms than did bigger

houses. Some early nineteenth-century houses had wide floorboards near the walls of each room and narrower floorboards in the center, where

the floors were usually carpeted. Many of the biggest early niaeteenth-century houses but very few small houses from that period had some floors that were made of materials that were considerably more expensive.

PDF compression OCR web optimization using a watermarked evaluation copy of CVISION PDFCompressor  

104.

@

Speaker: Contemporary business firms need to

rccagnize that avoiding social responsibility

leads to the gradual erosion of power. This is

105. Party spokesperson: The opposition party's proposal to stimulate economic activity in the

not use power in a manner which society considers responsible will tend to lose it." The

province by refunding 96O0 million in provincial taxes to taxpayers, who could be expected to spend the money, envisions an illusory benefit. Since the province's budget is required to be in balance, either new taxes

law's application to human institutions certainly stands confirmed by history. Though the "long rurt''may require decades or even centuries in some instances, society ultimately acts to reduce power when sociefy thinks it is not being used responsibly. Therefore, a business that wishes to retain its power as long as it can must act responsibly.

would be needed to make up the shortfall, in which case the purpose of the refund would be defeated, or else workers for the province would be dismissed. So either the province's taxpayers or its workers, who are also residents of the province, will have the $600 million to spend, but there can be no resulting net increase in spending to stimulate the

Davis and Blomstromt Iron Law of Responsibility: "In the long run, those who do

Which one of the following statem€nts, if true, most weakens the speaker's argument?

(A)

Government institutions are as subject to the Iron Law of Responsibility as business

(B)

Public relations ptograms can caus€ society to

(C) (D) (E)

institutions.

consider an institution socially responsible even when it is not. The power of some institutions erodes more

slowly than the power of others, whether they are socially responsible or not. Since no institution is eternal, every business will eventually fail. Some businesses that have used power in socially responsible ways have lost it.

* * The conclusion about whether there would be a resulting net increase in spending would not follow if * the (A) taxpayers of the province would spend outside * provincets economy.

the province at least $300 million of any $600

tB) (C) (D)

(E)

million refunded to them taxpayers ofthe province would receive any refund in partial payments during the year rather than in a lump sum province could assess new taxes in a way that

would avoid angering taxpayers province could, instead of refunding the money, stimulate its economy by redirecting its spending to use the $600 million for construction projects creating jobs around the province province could keep its workers and use them more effectively, with a resulting savings of $600 million in its out-of-province erpenditures

Source: PrcpTest 22, Section

4, Questions

9

and 24

265

g, F ce ' -Iif;

It

-fr

o

sl

rfi

o it

ll-r

SQ

PDF compression OCR web optimization using a watermarked evaluation copy of CVISION PDFCompressor  

Mqstery Proctice

ISAT

106, Members of the Amazonian Akabe people commonly take an early-morning drink of a tea made from the leaves of a forest plant' Although they greatly enjoy this drink, at darvn they drink it only in small amounts. Anthropologists hypothesize that since this tea is extraordinarily high in caft'eine, the erplanation for the Akabe's not drinking more of it at dalvn is

@

intake would destroy the that high caffeine surefootedness that their daily tasks require.

people in the locality who smoke cigarettes.

Which one of the following, if true, most helps to strengthen the argument?

Which one of the following, if true, most seriously calls the anthropologists' explanation into question?

(A) (B)

* * * *

tr

rllbo ,

El

o, Utl

|r

o

&

I

l

v

I

bo

(t J

(C) (D) (E) LA7.

(A)

The drink is full of nutrients otherwise absent from the Akabe diet. The Akabe also drink the tea in the evening, after their dayt work is done. The leaves used for the tea contain a soluble

(B)

narcotic.

Akabe children are introduced to the tea in only a weak form. When celebrating, the Akabe drink the tea in large quantities,

(C) (D)

Raising the tax rate on essential goods-a traditional means of increasing government revenues-invariably

turns low- and middle-income taxpayers against the government, Hence government officials have proposed adding a new tax on purchases of luxury items such as yachts, private planes, jewels, and furs. The officials claim that this tax will result in a substantial increase in government revenues while affecting only the wealthy individuals and corporations who can afford to purchase such items. The answer to which one of the following questions would be the most relevant in evaluating the accuracy of the government officials' prediction?

(A)

Will luxury goods be taxed at a higher rate

(B)

the revenues generated by the proposed Will tax be cornparable to those that are currently

(C) (D) (E)

initiated an antismoking advertising campaign in local newspapers, which it financed by imposing a tax on cigarettes of 20 cents per pack One year later, the number of people in the locality who smoke cigarettes had deciined by 3 percent. Clearly, what was said in the advertisements had an effect, although a small olle, on the number of

108. One year ago a local government

than that at which essential goods are currently taxed?

being generated by taxes on essential goods? Will sales of the luxury items subject to the proposed tax occur at current rates once the proposed tax on luxury items has been

(E)

of centipedes that are 414 million years old. These fossils are at least 20 million years older than the earliest Iand-dwelling animals previously identified. The paleontologists are confident that these centipedes lived on land, even though the fossilized centipedes were discovered in rock that also contained fossilized remains of animals known to be water-dwelling.

109. Paleontologists have discovered fossils

@ @

The paleontologists'view would be LBAST supported by the truth of which one of the following?

(A) (B) (C)

Passed?

Will the proposed tax on luxury items rvin support for the government in the eyes of

low- and middle-income taxpayers? Will purchases of luxury iterns by corporations account fbr more of the revenue generated by the proposed tax than will purchases of luxury items by wealthy individuals?

Residents of the localit,v have not ilcreased their use of other tobacco products such as snuff and chen'ing tobacco since the campaign went into effect. A substantial number of cigarette smokers in the locality who did not quit smoking during the campaign nolt smoke less than they did before it began. Admissions to the local hospital for chronic respiratory ailments were down by l5 percent one year after the campaign began. Merchants in the locality responded to the local tax by reducing the price at which they sold cigarettes by 20 cents per pack. Smokers in the locality had incomes that on average were 25 percent lower than those of nonsmokers.

(D) (E)

legs of the The fossilized centipedes were suited particularly to being a means of locomotion on land, Ail of the centipedes that had previously been discovered were land dwellers. The rock in which the fossilized centipedes were found was formed from mud flats that were occasionally covered by river water. Fossils of the earliest land-dwelling animals that had previously been identified rvere found in rock that did not contain fossilized remains of water-dwelling animals. Fossils of spiders with respiratory sysrems adapted only to breathing air were found in the same rock as the centipede fossils.

Sources: PrcpTest PrcpTest tuepTest tuepTest

266

2, 7,

70,

Section Section Section Section

2, 3, 3, 7,

Question 7Z Question Question 2 Question g

PDF compression OCR web optimization using a watermarked evaluation copy of CVISION PDFCompressor  

Str e n gth e n /Wea ke n on the city-run public buses in Greenville are subsidized by city tax reyenues, but among the beneficiaries of the low fares are many people who commute from outside the city to jobs in Greenville. Some city councillors argue that city taxes should be used primarily to benefit &e people who pay them, and therefore that bus fares should be raised enough

I 10. Fares

@

to cover the cost of the service. Each of the following, if true, would weaken the argument advanced by the city councilors EXCEPT:

(A) (B)

(C)

(D) (E)

Many businesses whose presence in the city is beneficial to the city's taxpayers would relocate outside the city if public-transit fares were more expensive.

By providing commuters with economic incentives to drive to work, higher transit fares would worsen air pollution in Greenville and increase the cost of maintaining the city's streets. Increasing transit fares would disadvantage those residents of the city whose low incomes make them exempt from city taxes, and all city councilors agree that these residents should be able to take advantage of city-run services. Voters in the city, many of whom benefit from the low transit fares, are strongly opposed to increasing local taxes. People who work in Greenville and earn wages above the nationally mandated minimum all pay the city wage tax of 5 percent,

I

1

eu esti o n s

1. There is relatively little room for growth in the

overall carpet market, which is tied to the size of the population. Most who purchase carpet do so only once or twice, first in their twenties or thirties, and then perhaps again in their fifties or sixties. Thus as the population ages, companies producing carpet will be able to gain market share in the carpet market only through purchasing competitors, and not through more aggressive marketing.

Which one of the following, if true, casts the most doubt on the conclusion above?

(A) (B)

(C) (D) (E)

Most of the major carpet producers market other floor coverings as well. Most established carpet producers market several different Ltrand names and varieties, and there is no remaining niche in the market for new brands to fiIl. Two of the three mergers in the industry's last ten years led to a decline in profits and

revenues for the newly merged companies. Price reductions, achieved by cost-cutting in production, by some of the dominant firms in carpet the causing other producers to leave themarket marketare altogether. The carpet market is unlike most markets in that consumers are becoming increasingly resistant to new patterns and styles.

* * * *

F a

oe II

n

ot I

n o

gl

{'r

o =q I

Sourceg' PrepTest 10, Sectlon Preplest 10, Section

Qsestlon 9 t, 4, Questlon g

1tY

267

PDF compression OCR web optimization using a watermarked evaluation copy of CVISION PDFCompressor  

rt2.

@ @

The number of afucraft collisions on the growrd is increasing because of the substantial increase in the number of fligtttt operated by the airlines. Many of the fatalities that occur in such collisions are caused not by the collision itself, but by an inherent flaw in the cabin design of most aircraft, in which seats' by restricting access to emergency exits, impede escaPe' Therefore, to redrtce the total number of fatalities that result annually from such collisions, the airlines should be required to remove all seats that restrict access to emergenry exits'

113.

@

A translation invariably reflects the writing style ofthe translator. Sometimes when a long document needs to be translated quickly, several translators are put to work on the job, each assigned to translate part of the document. In these cases, the result is usually a translation marked by different and often incompatible

writing sryles. Certain computer programs for

language translation that work without the intervention of human translators can finish the job faster than human translators and produce a stylistically uniform translation with an 80 percent accuracy rate. Therefore, when a long document needs to be translated quickly, it is better to use a comPuter

Which one of the following proposals, if implemented together with the proposal made in the pasug*, would improve the prospects for achieving

* * * *

translation program than human translators.

Which one of the following issues would be LEAST importaat to resolve in evaluating the argument?

ihe stat"d objective of reducing fatalities?

(A)

(B)

n0 g

.t'

(c)

I*

snl

(D)

tg

g'

r,8', .:..:

E.

(E)

TU

IJ rT

The airlines should be required, when buying new planes, to buY onlY Planes with unrestricted access to emergency exits. The airlines should not be permitted to increase further the number of flights in order to offset the decrease in the number of seats on each aircraft.

J

(B)

(C)

authorities should be required to Airport stteamiine their passenger check-in

procedures to accommodate the increased number of passengers served by the airlines' Airport authorities should be required to refine security precautions by making them less conspicuous without making them less effective, The airlines should not be allowed to increase the tickei price for each passenger to offset the decrease in the number of seats on each

aircraft,

bo Q

(A)

(D) (E)

114.

@

whether the problem of stylistic variety in human translation could be solved by giving stylistic guidelines to human translators whether numerical comparisons of the accuracy of translations can reasonably be made

whether computer translation programs, like human translators, each have their own distinct writing style whether the computer translation contains errors of grammar and usage that drastically alter the meaning of the text how the accuracy rate of computer translation programs compares with that of human translators in relation to the users'needs

Mr

It

Q:

Seventy to ninetyyears ago, however, children were punished for using their left hands to eat

is almost impossible to find a person between the ages of 85 and 90 who primarily uses the left hand.

or to write and were forced to use their right

hands. Q's response serves to counter any Bse by M of the evidence about 85 to 90 year olds in support of which one of the following hypotheses?

(A) tB)

(C) (D) (E)

Being born right-handed confers a survival advantage. Societal attitudes toward handedness differ at different times. Forcing a p€rson to switch from a preferred hand is harmless. Handedness is a product of both genetic predisposition and social pressures. Physical habits learned in school often persist

in old age.

Sources: PrepTest 9, Section 2, Question lO PrcpTest 9, Section 4' Question 8 *eplest 5, Sectlon Questlan t5

l,

268

PDF compression OCR web optimization using a watermarked evaluation copy of CVISION PDFCompressor  

I 15. A ftee's age can be determined by counting the annual growth rings in its trunk Each ring represents one year, and the ring's thickness reveals the relative

amount of rainfall that year. Archaeologists successfrrlly used annual rings to determine the relative ages of ancient tombs atPazyryk Each tomb was constructed from freshly cut logs, and the tombs'

builders were constrained by tradition to use only logs from trees growing in the sacred PazyrykValley. Which one of the following, if true, contributes most to an explanation of the archaeologists'success in using annual rings to establish the relative ages of the tombs at the Paryryk site?

(A)

(B)

(C) (D) (E)

The Paryryk tombs were all robbed during ancient times, but breakage of the tombs' seals allowed the seepage of water, which soon froze permanently, thereby preserving the

* * * *

tombs' remaining artifacts.

The PazyrykValle , surrounded by extremely high mountains, has a distinctive yearly pattern of rainfall, and so trees growing in the PazyrykValleyhave annual rings that are quite distinct from trees growing in nearby valleys. Each log in the Pazyryk tombs has among its rkgs a distinctive sequence of twelve annual rings representing six &ought years followed by three rainyyears and tltree more drought years. The archaeologists determined that the youngest tree used in any of the tombs was 90 years old and that the oldest tree was 450 years old, All of the Pazyryk tombs contained cultural artifacts that can be dated to roughly 2300 years ago,

F c

qs.

a

al I

7

o al o

UT

=. =Q

Source: PtepTest

5, Scatlon

,

eueeuon 19

{l19

26s

View more...

Comments

Copyright ©2017 KUPDF Inc.
SUPPORT KUPDF